You are on page 1of 135

SECTION A (50 MARKS)

1.

The variables x and y are connected by the equation x = ayb, where a and b are
constants. Pairs of corresponding values of x and y are given in the table below.
y
x

0
0

1
2

2
16

3
c

Find the values of a, b and c.

MATHEMATICS
(Secondary Two)

2.

(2 hours)

3.

4.

6 printed pages, including this cover page and answers sheet

[1]
[1]

Express x in terms of a, b and c.


(a) 2ax b = cx + 1
(b) x 2c a b
a

6.

[1]
[2]

Factorise completely
(a) xy + 5yz 2y
(b) 4p2 25q2

5.

[1]
[1]

Expand
(a) 3a(2b 5c + d)
(b) (2p q)(p + 3q)

Answer all questions in Section A AND Section B.


If the degree of accuracy is not specified in the question, and if the
answer is not exact, give the answer to three significant figures.
Give answers in degrees to one decimal place.
The number of marks is given in brackets [ ] at the end of each
question or part question.
The total number of marks for this paper is 100.
The use of an electronic calculator is expected, where appropriate.
Omission of essential working will result in loss of marks.
You are reminded of the need for clear presentation in your
answers.

In a class project selection, three class projects, namely, Project A, Project B and
Project C were presented. 90% of the class voted and the votes for these projects
were divided in the ratio 5:3:1 respectively. Given that 45 pupils voted, calculate
(a) the number who did not vote.
(b) the number of votes in favour of Project A.

End-Of-Year Examination 2014

[3]

The perimeter of a rectangular plot of land is 42 m. If


of its length, find the dimensions of the plot of land.

[2]
[2]
1
1
of its width is equal to
4
3

[3]

1/6

7.

Solve the following


(a) (3x + 2)(x 7) = 0
(b) 4x2 8x = 0

y
[2]
[3]
3

8.

A retailer prepares a mixture of coffee powder. He mixes 40 kg of type X coffee


powder, which he buys at $30 per kg, with 25 kg of type Y coffee powder, which
he buys at $20 per kg. At what price must he sell the mixture per kg so that he
earns 30% profit?
[3]

1
9.

Draw the graph of each of the following equations on the grid provided.
(a) x = 1

[1]

(b) y + 1 = 0

[1]

(c) y = 3 x

[2]

10. Solve the following simultaneous equations.


3x + 2y = 8
6x 3y = 12

[4]

2/6

11. It is given that US$1 = S$1.70. Complete the table, correct to the nearest cent.
US$
S$

13. The sides of a rectangle are formed by the lines x = 5, x = 2, y = 4 and y = 3 .


(a) Write down the coordinates of the vertices of the rectangle.
(b) Calculate the numerical value of its area.

9
93.50

[4]
[2]

[3]
12. In the quadrilateral ABCD, point E is the point of intersection of the diagonals
AC and BD. Given that AE = BE, DE = CE and AD = BC,
(a) show that ADB BCA.
State the test of congruency.

[2]

(b) If ABE and CDE are similar, given also that CD = 4 cm, DE = 3 cm
and BD = 10 cm, calculate the length of AB.
D

14. There are a total of 40 students in a class. There are 25 boys and 15 of them are
bespectacled. 20% of the girls either do not wear spectacles or they wear contact
lenses to school. All the students are Chinese. On a particular day, Mr Tan
picked a student to be the leader. Calculate:
(a) the probability that the student picked is a boy wearing spectacles.
(b) the probability that the student picked is non Chinese.
(c) the probability that the student picked is a girl without spectacles.

[1]
[1]
[2]

[2]
C

3/6

SECTION B (50 MARKS)

17. An athlete runs a distance of 700 metres in 2 minutes and then, without stopping,
walks a further distance of 1 km at an average speed of 4 km/h. Calculate

15. (a) Express as a single fraction in its simplest form

2
5 .

x 1 2x 3

[2]

(b) Factorise completely 7x2 15x + 2.

[2]

(c) Using a suitable algebraic rule, calculate 1052 52.

[2]

(d) Given that y

ax 2
c , calculate the value of y if a = 1, b = 2, c = 4 and
b

x = 3.

[2]

16. The diagram below shows four lines. The equation of the line BC is y + 2x = 10
and the equation of the line CD is y = 2.
y

(a) the height of the cylinder.


(b) the total surface area of the cylinder.

[2]
[2]

19. ABCDEF is a regular hexagon.


(a) Calculate

[3]
[2]

(b) DE and AF are produced to meet at P.


[1]
Calculate PEF.

[2]
[2]

18. The volume of a solid cylinder is 198 cm3 and its circular base has a radius of
3 cm. Taking to be 3.142, calculate

(i) BAC
(ii) EAC

(a) her running speed in km/h.


(b) her average speed for the whole journey in km/h.

C
E

(a) Write down the equation of the line AB.


(b) Write down the equation of the line AD given that the gradient is 2.
(c) What is the area of the triangle EDC?

[1]
[2]
[3]

4/6

20. Answer the whole of this question on a sheet of graph paper.


The variables x and y are connected by the equation y = x2 5x + 3 and some
corresponding values are given in the following table.
x
y

0
3

1
1

2
a

3
3

4
b

Number of Books
Number of Pupils

5
3

(a) Calculate the values of a and b.

[2]

(b) Taking 2 cm to represent 1 unit on the x-axis and the y-axis, draw the graph
[2]
of y = x2 5x + 3 for the range 0 x 5.
(c) On the same graph, draw the graph of y = 1.5.

23. The table shows the number of books borrowed by a class of 40 pupils over a
semester.

[1]

(d) Estimate, to 1 decimal place, the solutions of y = x2 5x + 3 and y = 1.5


from the coordinates of their points of intersection.
[2]

0
1

1
4

2
8

(a) Find the Mode.


(b) Find the Median.
(c) Calculate the mean number of books read.

3
4

4
9

5
12

6
2
[1]
[2]
[1]

24. In a class of 40 pupils, 25 of them have been to China and 17 of them have been
to Japan. Find the smallest and largest possible number of pupils who have been
to both countries.
[4]

21. Given that = 1, 2, 3, , 25, draw labeled Venn diagrams to illustrate the
following sets, placing the elements in the appropriate regions and answer the
questions that follows:
(a) A = multiples of 3, B = multiples of 9. Find A B.
(b) C = multiples of 2, D = prime numbers. Find C D.

[2]
[2]

22. The time (in minutes) to complete a mathematics worksheet by 20 pupils are as
follows.
25
36

28
39

35
18

41
23

32
19

19
45

Draw a stem-and-leaf diagram for the above data

38
27

43
21

19
25

33
33
[3]

5/6

ANSWERS (dont peek till youve tried the question!)

14. (a) 3
8

1.

a = 2, b = 3, c = 54

15. (a)

2.

(a) 5

3.

(a) 6ab 15ac + 3ad

(b) 25

4.

(a) y(x + 5z 2) (b) (2p + 5q)(2p 5q)

5.

(a) x b 1
2a c

7.

(b) a + ab 2c

width = 9 m, length = 12 m
2
(a) x or 7 (b) x = 0 or 2
3

8.

$34/per kg

9.

3
10

(c) 11000

16. (a) y = 4

(b) y = 2x + 2

17. (a) 21 km/h

(b) 6 km/h

18. (a) 7.00 cm

(b) 189 cm2

19. (ai) 30

(aii) 60

(c) 8 units

(b) 60

20. (a) a = 3, b= 1(d) (0.3, 1.5) and (4.7, 1.5)


21. (a) A B = {9, 18}
(b) C D = {2, 3, 4, 5, 6, 7, 8, 10, 11, 12, 13, 14, 16, 17, 18, 19, 20, 22, 23, 24}
22. Stem Leaf
1
2
3
4

10. x = 0, y = 4
11.
US$
S$

9x 1
(b) (7x 1)(x 2)
( x 1)( 2 x 3)

(c)

(d) 8.5

(b) 2p2 + 5pq 3q2

6.

(b) 0

9
15.30

55
93.50

23. (a) 5

8,9,9,9
1,3,5,5,7,8
2,3,3,5,6,8,9
1,3,5
(b) 4

(c) 3.5

24. 2, 17

12. (a) AD = BC, DE = CE, EB = EA (given).


DE + EB = CE + EA, i.e. DB = CA

ADB BCA (SSS)


(b) AB 9 1 cm
3
13. (a) (5, 4), (2, 4), (2, 3) and (5, 3) (b) 49 units2

6/6

SECTION A
1.

If x = 2 and y = 1,

x = ayb
2 = a(1)b
a = 2

If x is 16 and y is 2, x = ayb
16 = 2(2) b
= 2b

MATHEMATICS
(Secondary Two)

8 = 2b
23 = 2b
b = 3
If x = c and y = 3,

End-Of-Year Examination 2014


I
Worked Solutions
(Dont peek till youve tried the question!)

2.

x
c
c
c

=
=
=
=

2y3
2(3)3
2(27)
54

(a) 90% 45 pupils


1%

pupils

10%

10

= 5 pupils
there are 5 students who did not vote.
(b) 9 units 45 votes
1 unit

votes

5 units

5
= 25 votes

8 printed pages

there are 25 votes for Project A.

1/8

3.

(a) 3a(2b 5c + d) = 6ab 15ac + 3ad

Substitute x =

into (2),

(b) (2p q) (p + 3q) = 2p2 + 5pq 3p2


4.

5.

(a)

(b)

Using a2 b2 = (a b)(a + b)

Substitute y = 12 into (3), x + 12 = 21


x = 21 12
x = 9
The width of the plot of land is 9 m and its length is 12 m.

2ax b = cx + 1
2ax cx = b + 1
x(2a c) = b + 1
x =

6.

= 21
7y = 84
y =
y = 12

(a) xy + 5yz 2y = y(x + 5z 2)


(b) 4p2 25q2 = (2p)2 (5q)2
= (2p + 5q) (2p 5q)

=

x + 2c =
x + 2c =
x =

7.

a+b
a(a + b)
a2 + ab
a2 + ab 2c

Let the width of the plot of land be x and its length be y.



(1)
8.

x + x + y + y = 42
2x + 2y = 42
2(x + y) = 42
x+y =
x + y = 21
From (1),

(2)

=
=
4x = 3y
x =

+ y = 21

(3)

(a)

(3x + 2)(x 7)
3x + 2
3x
x
2
(b) 4x 8x = 0
4x(x 2) = 0
4x = 0
x = 0
Cost of X =
=
Cost of Y =
=
Total Cost =
=

= 0
= 0
= 2
=

or

or

x7 = 0
x = 7

x2 = 0
x = 2

$30 40
$1200
$20 25
$500
$1200 + 500
$1700

1700
30% Profit =
= $510
Total selling price = $1700 + 510
= $2210
Selling price for 1kg of the mixture = $2210 65
= $34

2/8

9.

10.
y

3x + 2y = 8
6x 3y = 12
1

2,

6x + 4y
(3) (2),
6x + 4y (6x 3y)
4y + 3y
7y

x=1
3

(1)
(2)

= 16

(3)

= 16 (12)
= 16 + 12
= 28

y =

y = 4
1

Substitute y = 4 into (3),

y=3x

6x + 4(4)
6x + 16
6x
6x
x

x
1

y = 1

16
16
16 16
0
0

x = 0 and y = 4

=
=
=
=
=

11.

US$1 = S$1.70
US$9 = S$1.70 9
= S$15.30
S$1.70 = US$1
S$1 = US$
S$93.50 = US$

.
.

93.50

= US$55
US$
S$

9
15.30

55
93.50

3/8

y
12. (a)

AD =
DE =
DE + EB =
Given EB =
i.e. DB =

BC
CE
CE + EA
EA,
CA

y=4

ADB BCA (SSS)


(b)

EB = 10 cm 3 cm
= 7 cm

x = 5

x=2
2

Ratios of corresponding sides are equal.

=
1
=
AB =

4
5

=
=

cm

13. (a) Coordinates of the vertices of the rectangle

(5, 4), (2, 4), (2, 3), (5, 3)


(b) Area of the rectangle = 7 7 (refer to diagram on the next page)
= 49 units2

y = 3

4/8

14. (a) Number of bespectacled boys = 15


Probability that the student picked is a boy wearing spectacles

SECTION B

15. (a)

(b) Number of non-Chinese students = 0


Probability = 0 (Impossible Event)

(b) 7x2 15x + 2 = (7x 1) (x 2)


(c) Number of girls = 40 25
= 15
Number of girls without spectacles =

15

= 12
Probability that the student picked is a girl without spectacles
=
=

(c) 1052 52 = (105 + 5) (105 5)


= 110(100)
= 11000
(d)

y =

=
=

Using a2 b2 = (a + b)(a b)

= 4.5 + 4
= 8.5

5/8

16. (a) Equation of line AB y = 4

17. (a)

(b) Gradient, m = 2, y-intercept, c = 2

Equation of line y = mx + c

Equation of line AD y = 2 + 2x

700 cm = 700 1000


= 0.7 km
2 minutes = 2 60
=
hour

(c) Sub y = 2 into equation of BC,


Her running speed =

2 + 2x = 10
x= 4
C = (4, 2)

y = 2 + 2x
y + 2x = 10

(1)
(2)

Sub (1) into (2),


2 + 2x + 2x = 10
4x = 8
x = 2
Sub x = 2 into (1),

= 21 km/h
(b) Total time taken = (2 + 15) minutes
= 17 minutes
=
hour
Total distance = 1 km + 0.7 km
= 1.7 km
Average speed =

y = 2+4
y = 6

= 6 km/h

E = (2, 6)
Base of triangle EDC = DC
= 4 units
Height of triangle EDC = Distance from E to line DC
= 4 units
area of the triangle EDC =
=

Base Height
4 4

= 8 units2

18. (a) Volume of Cylinder


198
198
198 28.278
h
(b)

=
=
=
=

Total surface area =


=
=
=

3.142 3 3 h
28.278
h
7.00 cm (3 sig.fig)
2
2
(2 3.142 3 7) + (2 3.142 3 3)
131.964 + 56.556
188.52cm3
189 cm3 (3 sig. fig)

6/8

20. (a)

19. (a) (i) Each interior angles in a 6-sided polygon


=

x
y

=
= 120
BAC =
=

BAC is an
isosceles

Each interior angle


in n-sided polygon
=

0
3

1
1

2
3

3
3

4
1

5
3

(b)

= 30
(ii) EAC = 120 30
= 60

30

(b) PFE = PEF


= 180 120
= 60 (adjacent angles on same line)
PEF = 180 PFE PEF
= 180 60 60
= 60 (sum of angles of triangle)

(c) Refer to diagram


(d) Coordinates of intersection = (0.3, 1.5) and (4.7, 1.5)

7/8

21. (a)

(b)

A = {3, 6, 9, 12, 15, 18, 21, 24}


B = {9, 18}
A B = {9, 18}

24.

= {pupils in the class}


C = {pupils who have been to China}
J = {pupils who have been to Japan}

C = {2, 4, 6, 8, 10, 12, 14, 16, 18, 20, 22, 24}

Let n(C J) = x, the number of pupils who have been to both countries.

D = {2, 3, 5, 7, 11, 13, 17, 19, 23}

n(C J) is smallest when n(C J) = n( )

C D = {2, 3, 4, 5, 6, 7, 8, 10, 11, 12, 13, 14, 16, 17, 18, 19, 20, 22, 23, 24}
22.
Stem
1
2
3
4

Leaf
8, 9, 9, 9
1, 3, 5, 5, 7, 8
2, 3, 3, 5, 6, 8, 9
1, 3, 5

n(C J) is largest when J C


Then n(C J) = n(J)
x = 17

23. (a) Mode = 5 books


(b) Median = 4
(c) Mean =
=

25 x + x + 17 x = 40
42 x = 40
x = 2
The smallest possible number of pupils who have been to both countries is 2.

J
25 x

x 17 x

= 3.5

8/8

SECTION A (50 MARKS)


1.

The following is a set of nine whole numbers


5, 0, 3, 4, 8, 2, 1, 3, 1.
If the median is exactly 2.5 when another number is added, find the largest
possible value for the tenth number.
[2]

MATHEMATICS
(Secondary Two)

2.

Through what angle does the minute hand of a clock turn from 1230 to 1319?
[2]

3.

On a map, a length of 5 cm represents an actual distance of 1 km. Find the area on


the map, in cm2, which represents an actual area of 8 km2.
[2]

4.

Expand and simplify

End-Of-Year Examination 2014


(2 hours)

( x 3)(2 x 4) 3( x 5)( x 1)

II
Answer all questions in Section A AND Section B.
If the degree of accuracy is not specified in the question, and if the
answer is not exact, give the answer to three significant figures.
Give answers in degrees to one decimal place.
The number of marks is given in brackets [ ] at the end of each
question or part question.
The total number of marks for this paper is 100.
The use of an electronic calculator is expected, where appropriate.
Omission of essential working will result in loss of marks.
You are reminded of the need for clear presentation in your
answers.

5.

Solve the following equation below.

3x 1
x3
8
2
5
6.

[3]

[3]

(a) Make d the subject of the formula:


n
S [2a ( n 1)d ]
2

[2]

(b) Find the value of S when a = 3, n = 4 and d = 1

[1]

6 printed pages, including this cover page and answers sheet

1/6

7.

ABCDE is a pentagon with A = B = C = D = 2x, E = x and


AB = BC = CD.

11. The graph below is y = 3 + 2x x2.

(a) How many lines of symmetry does the pentagon ABCDE have?

[2]

(b) Calculate the value of x.

[2]

8.

Use the elimination method to solve the following pair of simultaneous equation
below.
7x + 4y = 22
4x + 5y = 37
[4]

9.

Factorise the following.


(a) 3x(a 5b) 5y(a 5b)

[1]

(b) p(4m n) 2p(m + 2n)

[3]

(a) Find the coordinates of the two points which lie on the x-axis.

[2]

(b) Find the coordinates of its maximum point.

[2]

x
12. The table below shows the sum of the scores of spinning a circular card spinner
twice. Complete the table below.

10. Given that is the universal set and A B , shade the set A B in the venn
diagram below.
[2]

2
3

B
+
Score 1
Score 2
Score 3

Score 1
2
3
4

Score 2
3

Score 3
4
5

From the above table, find the probability that the sum of the resulting scores is
(a) 2
[1]
(b) odd
[1]
(c) at least 4
[1]

2/6

13. It takes 12 men to make 9 cupboards in 4 days. How long will it take 4 men to
make 24 cupboards?
[3]

14. The diagrams show two ways of packaging 4 identical balls. The radius of each
ball is 7 cm.
Diagram I shows a closed rectangular box with a square base.
Each ball touches the top, the bottom and two sides of the box.
Each ball also touches 2 other balls.
Diagram II shows a closed cylinder.
The balls touch the ends and the side of the cylinder.
(Take

SECTION B (50 MARKS)


16. Simplify the following algebraic expression below, leaving your answer as a
fraction with positive indices.

(4 x) 3 y 4
5 xy 2
14 x 2

[
]
3ab 4
21(ab) 3 15 y 3

[3]

17. A six-faced die was thrown 20 times. The table shows the number of times that
each possible score occurred.

22
)
7

Score
Frequency

1
2

2
x

3
4

4
4

5
4

6
y

(a) Find the value of x + y

[2]

(b) If the model score was 2, find the largest possible value of y

[2]

(d) The mean score of all 20 throws was 3.5, find the values x and y

[3]

18. The equation of a straight line is 6x + 2y = 7.

(a) Write down the dimensions of the rectangular box.


(b) Calculate total surface area of the outside of the cylinder.
(c) Calculate the value of

volume of the cylinder


volume of the box

[2]
[3]

(a) Find the gradient of the line.

[1]

(b) Find the coordinates of the point where the line cuts the x-axis.

[2]

(c) Another line y = mx + c is parallel to the given line and passes through the
point (3, 5). Find the value of c.
[2]

[2]

15. If each pupil in a class sends a Christmas card to every classmate, the total
number of cards sent by all pupils will be 870. Find the number of pupils in the
class.
[4]

3/6

19. Answer the whole of this question on a sheet of graph paper.


The table below shows the frequency distribution of the number of pupils with
siblings (brothers and sisters) in a class of 36.
No. of siblings (x)
No. of pupils (f)

0
3

1
7

2
10

3
6

4
5

5
3

6
1

7
1

21. The line x + ky = 18 has a gradient of

1.
6

(a) Find the value of k.

[1]

(b) What is the y-intercept of this line?

[1]

(c) Find the value of x when y = 0.

[1]

(a) Find, from the frequency distribution table


(i) the mean,
(ii) the median and
(iii) the mode of the distribution

[1]
[1]
[1]

(b) Draw a Histogram to represent the table above, using a scale of 2 cm


to represent 1 unit for both x and y axes.

[3]

20. Pyramid VPQRS has a square base of sides 6 cm. The volume of the pyramid is
48cm3. Find
V

22. (a) A straight pole measuring 5 m leans against a vertical wall. The foot of the
pole is 1 m away from the base of the wall. Find the height of the wall from
the ground to the tip of the pole touching the wall (correct to 1 decimal
place).
[2]
(b) Five sticks of various lengths are tied together to form a structure as shown
in the diagram. Given that ADB = 90 and BCD = 90, find the length
of the stick labeled x (correct to 1 decimal place).
[4]

10 cm

S
N

17 cm

6 cm
(a) the height,
(b) VQ and
(c) the total surface area of the pyramid.

x cm

6 cm
Q
[1]
[2]
[3]

8 cm

4/6

23. The diagram shows two containers A and B. A is empty and it is made up of a
hemisphere of radius 6 cm fixed to a cylinder of radius 6 cm and height 35 cm.
B is completely filled with water and it is a cone of radius 13 cm and height
20 cm.

24. Answer the whole of this question on a sheet of graph paper.


This table gives the x and y coordinates of some points which lie on a curve.
x
y

6 cm

13 cm

2
5

1
2

0
0

1
1

2
1

3
0

4
2

5
5

(a) Taking 2 cm to represent 1 unit on each axis, and the values of x and y from
2 to 5, plot these points and draw a smooth curve through them.
[3]
(b) Write down the equation of the line of symmetry of this curve.

35 cm

20 cm

[1]

(c) The values of x and y are related by the equation y = Ax2 + Bx. Calculate the
values of A and B respectively.
[3]
Find the values of x which satisfy the equation Ax2 + Bx = 0.

[1]

B
A
(a) Given that A is an open container, find the total external surface area of A.
[2]
(b) If all the water in B is poured into A, find the depth of water in A.

[4]

5/6

ANSWERS (dont peek till youve tried the question!)

12. (a)

1
9

(b)

5
9

(c)

2
3

13. 32 days

1.

2.

294

3.

200 cm2

4.

x2 14x + 3

5.

6.

(a) d

14. (a) 28 cm by 28 cm by 14 cm
11
(c)
(b) 2772 cm2
14
15. x = 30
5 2
16. 96 y a
b

2( S an)
n(n 1)

(b) 18
7.

(a) 1

(b) 60

8.

x = 2, y = 9

9.

(a) (3x 5y)(a 5b)


(b) p(2m 5n)

10.

17. (a) x + y = 6

(b) 1

(c) x = 4, y = 2

18. (a) 3

(b) ( 7 ,0)
6

(c) c = 14

19. (ai) 2.58

(aii) 2

(aiii)2

20. (a) 4 cm

(b) 5.83 cm

(c) 96 cm2

21. (a) 6

(b) 3

(c) 18

22. (a) 4.9 m

(b) 11.2 m
2

AA

B
B

23. (a) 1550 cm

(b) 33.3 cm

24. (b) x = 1.5

(c) A =

3
1
, B = , 0 and 3
2
2

11. (a) (3, 0) & (1, 0)


(b) (1, 4)

6/6

SECTION A
1.

Numbers: 0, 1, 1, 2, 3, 3, 4, 5, 8
Medium of the nine numbers 3

To find median arrange the numbers in


ascending order

If the medium is 2.5, the tenth number has to be 3, i.e. 0, 1 or 2.


Largest possible value for the tenth number = 2
2.

MATHEMATICS
(Secondary Two)

49 min 49 6
= 294
3.

Map
Actual
5 cm : 1 km
1 cm : 0.2 km
(1 cm)2 : (0.2 km)2
1 cm2 : 0.04 km2
200 cm2 : 8 km2
The area on the map is 200 cm2.

4.

(x 3)(2x + 4) 3(x + 5)(x 1) = (2x2 + 4x 6x 12) 3(x2 x + 5x 5)


= (2x2 2x 12) 3(x2 + 4x 5)
= 2x2 2x 12 3x2 12x + 15
= x2 14x + 3

End-Of-Year Examination 2014


II
Worked Solutions

12.30 pm to 1.19 pm = 49 minutes


1 min
= 6

(Dont peek till youve tried the question!)


5.

8 =

=
=

8 printed pages

5(3x 17)
15x 85
13x
x

=
=
=
=

2(x + 3)
2x + 6
91
7

1/8

6.

(a)

S =

S =

8.

= 2
= d (n 1)

(2) 4:
4(4x + 5y) = 4(37)
16x + 20y = 148 (4)

= d (n 1)
d =

d =

(4) (3)
(16x + 20y) (35x + 20y)
19x
x
Substitute x = 2 into (1),
7(2) + 4y
14 + 4y
4y
y
x = 2, y = 9

d =
d =
(b) Substitute a = 3, n = 4 and d = 1 into equation,
1 =
1 =
1
2S 24
2S
S
7.

=
= 12
= 36
= 18

9.
Each interior angle in n-sided

polygon =

(a) Line(s) of symmetry: 1


(b)

(1)
(2)

(1) 5:
5(7x + 4y) = 5(22)
35x + 20y = 110 (3)

= 2

7x + 4y = 22
4x + 5y = 37

Sum of interior angles in pentagon = (5 3) 180


4(2x) + x = 540
9x = 540
x = 60

= 148 110
= 38
= 2
=
=
=
=

22
22
36
9

(a) 3x(a 5b) 5y(a 5b) = (3x 5b)(a 5b)


(b) p(4m n) 2p(m + 2n) = 4mp np 2mp 4np
= 2mp 5np
= p(2m 5n)

10. A B

AA

B
B

2/8

11. (a) Substitute y = 0 into equation,


x2 + 2x + 3 = 0
When the curve cuts at xx2 2x 3 = 0
axis, y-coordinate is 0.
(x + 1)(x 3) = 0
x+1 = 0
or
x3 = 0
x = 1
x = 3
The coordinates are (1, 0) and (3, 0)
=1
(b) Line of symmetry of the curve x =
Substitute x = 1 into equation,
y = 3 + 2(1) (1)2
y = 3+21
y = 4
Coordinates of its maximum point is (1, 4)
12.
+
Score 1
Score 2
Score 3

Score 1
2
3
4

Score 2
3
2+2 = 4
5

14. (a) Width of the box = 14 2


= 28 cm
Length of the box = 14 2
= 28 cm
Height of the box = 14 1
= 14 cm
(b) Surface area of cylinder =
=
=
=

2
+2 2
2 7 (14 4) + (2
2464 + 308
2772 cm2

(c) Volume of cylinder =

=
Volume of box

Score 3
4
5
3+3 = 9

72)

72 (14 4)

= 8624 cm3
= 28 28 14
= 10976 cm3
=
=

(a) P (the sum of the resulting score is 2) =


(b) P (the sum of the resulting score is odd) =
(c) P (the sum of the resulting score is at least 4) =
=
13. 12men
12men

9 cupboards
1 cupboard

12men

24 cupboard

24 = 10 days

4men

24 cupboard

10 3 = 32 days

4 days
day

15. Let x be the number of pupils in the class.


x(x + 1) = 870
x2 x = 870
2
x x 870 = 0
(x + 29)(x 30) = 0
x + 29 = 0
or
x = 29 (rejected)

x 30 = 0
x = 30

There are 30 pupils in the class

3/8

SECTION B

16.

x+y=6
x + 3y = 10

=
=
17. (a)

Total frequency
2+x+4+4+y
x+y
x+y

=
=
=
=

Modal score
= the score that occurs
most often

20
20
20 14
6

(b)

Since the modal score is 2, x must be > 4,


Largest possible value of y = 1 since x + y = 6

(c)

Mean =
= 3.5
= 3.5
= 3.5
25 + x + 3y = 35
x + 3y = 10

(1)
(2)

(2) (1),
(x + 3y) (x + y) = 10 6
2y = 4
y = 2
Substitute y = 2 into (1),
x+2 = 6
x = 4
Equation of the line
x = 4, y = 2
y = mx + c
where
m is the gradient and c
18. (a)
6x + 2y = 7
y-intercept.
is
the
2y = 6x + 7
y =
y = 3x + 3
Gradient of line = 3
(b) Substitute y = 0 into equation,
0 = 3x + 3
3x = 3

When the line cuts at x-axis,


y-coordinate is 0.

x = 3
x =
Coordinates of the point: ( , 0)
(c) Substitute x = 3, y = 5 and m = 3 into y = mx + c,
5 = (3)(3) + c
When the lines are parallel,
5 = 9 + c
gradients are the same.
c = 14

4/8

19. (a) (i) Mean =

(b)

2.58 (3 sig. fig.)


(ii) Median = mean of 18th and 19th values
=
=2
(iii) Mode of distribution = 2

5/8

20. (a)

Volume =

Base area Height

48 = 6 6 Height
48 = 12 Height
Height = 48 12
Using Pythagoras
= 4 cm
theorem
(b) SQ2 = 62 + 62
= 72
SQ = 72
Using QN2 + VN2 = VQ2,
72 2 + 42 = VQ2
18 + 16 = VQ2
VQ2 = 34
VQ = 5.83 cm
The length of VQ is 5.83 cm.
(c) Perpendicular height from V to QR = 4
3
= 5 cm
Total surface area of the pyramid
= 4 Area of VQR + Area of PQRS
= 4 6 5 + 6 6
= 60 + 37
= 60 + 36
= 96 cm2
Equation of the line
y = mx + c
21. (a) x + ky = 18
where
m is the gradient and
+y=
c is the y-intercept.

y=

(b) Substitute k = 6 into ,


3
y-intercept of the line is 3.
(c) Substitute y = 0 and k = 6 into x + ky = 18,
x + 0 = 18
Using Pythagoras theorem
x = 18
22. (a) Using a2 + b2
1 2 + b2
1 + b2
b2
b

=
=
=
=
=

c2,
52
25
24
4.9 (1 d.p.)

The height of the wall from the ground is 4.9 m.


(b) Using a2 + b2
102 + BD2
100 + AD2
AD2
AD

=
=
=
=

c2,
172
289
189
13.75 (2 d.p.)

x2 + 82 = 13.752
x2 + 64 = 189.06
x2 = 125.06
11.2 cm (1 d.p.)
x

=
k = 6

6/8

23. (a) Surface area of hemisphere =

4 r2

24. (a)

62

= 2

= 2 36
226.29 cm2
Curved surface area of cylinder = 2
= 2 6 35
= 1320 cm2
Total external surface area of A = 1320 + 226.29
= 1546.3 cm2
1550 cm2 (3 sig. fig)
(b) Volume of B =

132

20

169

20

3540.95 (2 d.p.)
Volume of hemisphere =

= 63
= 216
452.57 cm3
Volume of water in the cylinder part of container A
3540.95 452.57 = 3088.38 cm3
2
3088.38 cm3 =
3088.38 =
62 height
=

36

height


Height = 3088.38
27.30 cm (2 d.p.)
Depth of water in A = 27.3 + 6
= 33.3 cm

7/8

(b) Line of symmetry: x = 1.5


(c) Substitute x = 4 and y = 2 into y = Ax2 + Bx,
2 = A(4)2 + B(4)
2 = 16A + 4B
Substitute x = 2 and y = 5 into y = Ax2 + Bx,
5 = A(2)2 + B(2)
5 = 4A 2B
2 = 16A + 4B
5 = 4A 2B
(2)

(1)
(2)

4,
4(4A 2B) = 4(5)
16A 8B = 20

(3)

(3) (1),
(16A 8B) (16A + 4B)
12B
B
B

=
=
=
=

20 2
18
18 12

Substitute B = into (2),


5 = 4A 2(
4A + 3 = 5
A =
A = , B =
(d) When y = 0,
x = 0, x = 3

8/8

SECTION A (50 MARKS)


1.

Given that y is directly proportional to x + 2, and that y = 10 when x = 3,


(a) express y in terms of x.
(b) Hence, find the value of y when x = 5.

2.

MATHEMATICS
(Secondary Two)

[2]
[1]

While shopping, Leslie comes across the following advertisement at a Fifa outlet
which is having a closing down sale.

CLOSING DOWN SALE


FINAL REDUCTION OF
25% on all items
HURRY, STOCKS ARE
CLEARING FAST!

End-Of-Year Examination 2014


(2 hours)

(a) Calculate the original price of a jacket if Leslie buys it at a price of $99.

III
Answer all questions in Section A AND Section B.
If the degree of accuracy is not specified in the question, and if the
answer is not exact, give the answer to three significant figures.
Give answers in degrees to one decimal place.
The number of marks is given in brackets [ ] at the end of each
question or part question.
The total number of marks for this paper is 100.
The use of an electronic calculator is expected, where appropriate.
Omission of essential working will result in loss of marks.
You are reminded of the need for clear presentation in your
answers.

7 printed pages, including this cover page and answers sheet

[1]
(b) If the original price of a pair of leather pants is $200, calculate its price
during the sale.
[1]

3.

4.

Given that w x

4x w
,
z

(a) find the value of w when x = 5 and z = 9,

[1]

(b) express x in terms of w and z.

[2]

(a) Simplify w2 (w + 1)(w 1).

[1]

(b) Hence find the value of 999982 99999 99997.

[1]

1/7

5.

Spencer is given the task of drawing a plan of our schools layout for the coming
Learners and Leaders Conference. He decides to use a length of 4 cm on the plan
to represent an actual distance of 16 m.
Calculate
(a) the scale of the plan in the form of 1 : n

9.

(a) Solve 2a 1 3a .
7
(b) Simplify the expression (2x 3)2 x(3 x).

[1]

(c) Express as a fraction in its simplest form

[3]

[1]

21
5 .

16 x 2 x 4

(b) the actual length of a classroom, in metres, given that its length on the plan
is 5.5 cm
[1]
(c) the actual area, in m2, of the lecture theatre represented by 89 cm2 on the
plan
[2]

6.

Solve the simultaneous equations


9 x 5 y 52

[2]

10. In the diagram, ABCD is a quadrilateral with BA parallel to CD. AC and BD meet
at X where CX = 9 cm and XA = 12 cm.
A

[3]
12 cm

4 x 3 y 27
D

7.

Solve the equation 2y2 5y = 12.

8.

In the diagram below ABCDEF is a hexagon. Calculate its smallest interior angle.
[3]

[3]

9 cm

(a) Name a triangle similar to triangle BXA.

(b) Write down the value of

0.5x

B
2.5x

6x

[1]

DX
.
BX

[1]

(c) Given that CD is 13 cm, calculate AB.

[2]

4x C

11. (a) Factorise ab ac b + c.


1.5x

3.5x

[2]
2

(b) If x + y = 160 and xy = 50, find the value of 3(x + y) .

[2]

2/7

12. (a) It is given that = x : 1 x 15, x is positive integer. Sets A, B and C are
subsets of the universal set, . List the elements of
(i) A = {x : x + 3 10}
[1]
(ii) B = {x : x is a multiple of 3}
[1]
(iii) C = A B
[1]
(iv) Hence, shade in the Venn Diagram below the region representing the
set C.
[1]

13. By considering the number patterns given answer the following questions.
Line
1

34 16 = 1 5 (32 + 4)

44 16 = 2 6 (42 + 4)

54 16 = 3 7 (52 + 4)

64 16 = 4 8 (62 + 4)

..
.

(b) 44 students were given a choice to join a CCA of their choice. It is given that
B = students who choose basketball, S = students who choose soccer,
n(B S) = x, n(S) = 23 and n(B) = 30.
(i) Complete the Venn Diagram to illustrate the above information.

(a)

(b)

[1]

(a) Complete line 8 in the answer space above.

[1]

(b) Complete line n in the answer space above.

[2]

(c) Use your answer to part (b) and the information given below to find the three
prime factors of 50 609. [154 = 50 625]

(ii) Hence, find the number of students who play soccer only.
(iii) Describe the set B S in words.

[2]

[2]
[1]

3/7

SECTION B (50 MARKS)


14. (a) Given p 2q 1 , express q in terms of p and hence find the value of q
q 3
when p = 4.
[2]
(b) Solve 2 3( x 1) 3
x
5x

15. Factorise (2p q)(r s) + (4p + q)(s r) completely.

[3]

y2 1

[3]
17. A gardener finds that it takes 10 minutes less to cut his lawn with a new mower
which cuts at 12.5 m2/ min faster than his old one. The area of his lawn is 330
m2.
(a) Taking x to be the rate, in m2/ min, at which the new mower cuts, write
\
down an expression, in terms of x, for the time taken to cut the lawn with
the new mower.
[1]
(b) Write down an expression, in terms of x, for the time taken to cut the lawn
with the old mower.
(c) Hence form an equation in x and show that it reduces to
2x2 25x 825 = 0

(b) A line has equation y = 6x 5 and passes through the point (h, h + 10).
Calculate the value of h.
[1]
(c) The line y = 9 meets the y-axis at T. Write down the coordinates of T.

[1]

[2]

2
16. Simplify 2 y 3 y 5 6 y 15

y2 1

18. (a) A line passes through the point (0, 5) and has gradient 4. Write down the
equation of the line.
[1]

[1]

19. Mr Tan filled a hemispherical container completely with 10 litres of orange


juice. He then filled the orange juice into as many identical conical cups as
possible. The top radius of the cup is 4.2 cm and its height is 12 cm.
(Take to be 22 )
7
Calculate
(a) the number of conical cups Mr Tan could fill
[2]
(b) the volume of orange juice left in the container, giving your answer in
cubic centimeters
[2]
(c) the radius of the container, giving your answer correct to 2 decimal
places

[2]

20. The following are marks scored by 10 students in a Mathematics test marked out
of a total of 10:
3, 6, 4, 3, 5, 6, 7, 9, 6, 4
Find
(a) the modal mark
(b) the median mark
(c) the mean of this set of marks

[1]
[1]
[2]

[3]

(d) Solve this equation and find the time taken to cut the lawn with the new
mower.
[3]

4/7

22. The daily distance, in kilometres, traveled by a salesman in 30 days are shown
below in the stem-and-leaf diagram.

21. Answer the whole of this question on a sheet of graph paper.


The following shows a table of values for the equation y = x2 + 3x + 10.

x
y

2
0

0
10

2
12

(a) Copy and complete the given table

4
6

5
0

[2]

(b) Using a scale of 1cm for 1 unit for both the x and y axes, draw the graph of
[3]
y = x2 + 3x + 10 for 2 x 6.
(c) From the graph, state whether the curve has a minimum or maximum point
and its value.
[1]
2

(d) Use your graph to solve x + 3x = 6

[2]

1
2
3
4
5
6
7
8

2
0
0
3
0
4
6
5

4
1
0
6
1
5
7
8

9
3
1
6
2
7
8
9

8
7
6 9
8

(a) Construct a grouped frequency table for the data above using the class
intervals of 11 d 30, 31 d 50, 51 d 70 and 71 d 90, where d is
the distance traveled in kilometres.
[3]
(b) Using the table that you have constructed in (a) above,
(i) state the modal class
(ii) find the median distance travelled
(iii) find the estimated mean distance travelled.

[1]
[2]
[2]

(c) If a distance is chosen at random from 30 days, find the probability that the
distance traveled by the salesman is
(i) less than 35 km
(ii) between 45 km and 65 km
(iii) at least 85 km

[1]
[1]
[1]

5/7

ANSWERS (dont peek till youve tried the question!)

(bi)

1.

(a) y = 2(x + 2) (b) y = 6

2.

(a) $132

(b) $150

3.

(a) w = 6.5

(b) x

4.

(a) 1

(b) 1

5.

(a) 1 : 400

(b) 22 m

6.

x = 3, y = 5

7.

y = 1.5 or 4

8.

20

9.

(a)

w( z 1)
z4
(bii) 14
(c) 1424 m2

(biii)Students who choose both basketball and soccer as their


CCA.

13. (a) 104 16 8 12 102 4


(b)

n 2 4 16 n n 4 [(n 2) 2 4]

(c) 13 17 229
7
23

10. (a) DXC

(b) 5x2 15x + 9


(b)

3
4

(c)

5 x 41
16 x 2

(c) 17

1
3

2
14. (a) q 3 p 1 , q = 3.5
2

11. (a) (a 1)(b c) (b) 780


12. (ai) A = {1, 2, 3, 4, 5, 6, 7}
(aiii)C = {9, 12, 15}

(aii) B = {3, 6, 9, 12, 15}

12

15. 2(p + q)(s r)


y2 1
3( y 1)

17. (a)

330
min
x

18. (a) y = 4x 5

(aiv)

(b) x 7

p 2

16.

S
30 x x 23 x

(b)

330
min
x 12.5

(b) h = 3

(d) 12 min
(c) T = (0, 9)

19. (a) 45 cups

(b) 20.8 cm

20. (a) 6

(b) 5.5

(c) 16.84 cm
(c) 5.3

21. (a) When x = 1, y = 6; x = 1, y = 12; x = 3, y = 10; x = 6, y = 8


(c) Maximum point = (1.5, 12.25)
(d) x = 4.4 or 1.4

6/7

22. (a)
Distance Travelled (km)
11 d 30
31 d 50
51 d 70
71 d 90

Frequency
9
8
6
7

(bi) 11 d 30 (bii) 47.5 km

(biii) 49.5 km

(ci) 1

(cii) 3

(ciii) 2

10

15

7/7

SECTION A
1.

(a) y (x + 2)
y = k(x + 2), where k is a constant.

If y is directly proportional to x,
then y = kx, where k is a constant
and k 0.

Substitute y = 10 and x = 3 into y = k(x + 2),


10 = k(3 + 2)
10 = k(5)
k = 2

MATHEMATICS
(Secondary Two)

Sub k = 2 into y = k(x + 2),


y = 2(x + 2)
(b) Sub k = 2 and x = 5 into y = k(x + 2),

End-Of-Year Examination 2014


III
Worked Solutions
(Dont peek till youve tried the question!)

y = 2(5 + 2)
y = 2(3)
y = 6
2.

(a) 100% 25% = 75%


Original price of a jacket =

100

= $132
(b) Sale price = 75% $200
=

200

= $150

8 printed pages

1/8

3.

6.

(a) Substitute x = 5 and z = 9 into equation,



w5 =

(b)

w5
9(w 5)
9w 45
10w
w

=
=
=
=
=

wx
z(w x)
zw zx
zw + w
w(z + 1)

=
=
=
=
=

20 w
20 w
65
6.5

4x w
4x w
4x + zx
x(4 + z)

Using a2 b2 = (a b)(a + b)

(a) w2 (w + 1)(w 1) = w2 (w2 12)


= 12
=1

(1) 4:
4(9x 5y) = 4(52)
36x 20y = 208

(3)

(2) 9:
9(4x 3y) = 9(27)
36x 27y = 243

(4)

Substitute y = 5 into (1),


9x 5(5) = 52
9x + 25 = 52
9x = 27
x = 3

Using a2 b2 = (a b)(a + b)

(b) 999982 (99998 + 1)(99998 1) = 999982 (999982 12)


= 12
=1

(1)
(2)

(4) (3):
(36x 27y) (36x 20y) = 243 108
7y = 35
y = 5

x =
4.

9x 5y = 52
4x 3y = 27

x = 3, y = 5
7.

2y2 5y = 12
2y 5y 12 = 0
(2y + 3)(y 4) = 0
(2y + 3) = 0
or
2y = 3
y =
2

5.

(a)

4 cm
1 cm
1 cm
1: n

:
:
:
=

16 m
4m
400 cm
1: 400

(b)

1 cm : 400 cm
5.5 cm : 2200 cm
= 22 m

(c)

1 cm
(1 cm)2
1 cm2
89 cm2

:
:
:
:

(y 4) = 0
y = 4

4m
(4 m)2
16 m2
1424 m2

2/8

8.

Sum of interior angles in hexagon = (6 2)180


0.5x + 2.5x + 6x + 4x + 3.5 + 1.5x = 720
18x = 720
x = 40

Sum of interior angle


in n-sided polygon
= (n 2) 180

2a
2a
23a
a

=
=
=

11. (a) ab ac b + c = ab b ac + c
= b(a 1) c(a 1)
= (b c)(a 1)

2 2 3 3
(b) (2x 3)2 x(3 x) = 2
= (4x2 12x + 9) 3x + x2
= 5x2 15x + 9
(c)

Ratios of corresponding sides are equal

AB = 17

1 3
7(1 3a)
7 21a
7

=
=
=
=
=

(a)

9AB = 156

Smallest angle = 0.5x


= 20
9.

(c)

(b) 3(x + y)2 = 3(x2 + 2xy + y2)


= 3(x2 + y2 + 2xy)
= 3 160 2 50
= 3(160 + 100)
= 3(260)
= 780

Using (a + b)2 = a2 + 2ab + b2


Using a2 b2 = (a b)(a + b)

10. (a) BXA is similar to DXC.


(b)

Ratios of corresponding sides are equal

3/8

12. (a) (i) A = {1, 2, 3, 4, 5, 6, 7}

13. (a)

(8 + 2)2 16 = 8
Line 8 104 16 = 8

(8 + 4) [(8 + 2)2 + 4]
12 (102 + 4)

(b)

Line n(n + 2)4 16 = n

(n + 4) [(n + 2)2 + 4]

(ii) B = {3, 6, 9, 12, 15}


(iii) A = {8, 9, 10, 11, 12, 13, 14, 15}

(c)

C = A B
= {9, 12, 15}
(iv)

(n + 2)4 16
(n + 2)4
(n + 2)4
n+2
n

=
=
=
=
=

50609
50625
154
15
13

Substitute n = 13 into 50625 = n


50625
50625
50625
50625

=
=
=
=

13
13
13
13

(n + 4)

[(n + 2)2 + 4],

(13 + 4) [(13 + 2)2 + 4]


17 (152 + 4)
17 (225 + 4)
17 229

Prime factors of 50906 are 13, 17, and 229

(b) (i)

S
30 x x 23 x

(ii)

30 x + x + 23 x = 44
53 x = 44
x = 9
Number of students who play soccer only = 23 9
= 14

(iii) B S:
Students who choose both basketball and soccer as their CCA.

4/8

SECTION B
14. (a)

15. (2p +q)(r s) + (4p +q)(s r) = (2p +q)(r s) (4p + q)(r s)


= (r s)(2p q 4p q)
= (r s)( 2p 2q)
s r = (r s)
= 2(p + q)(r s)
= 2(p + q)(s r)

p =
P2 =
2

p (q 3) = 2q + 1
p2q 3p2 = 2q + 1

16.

p2q 2q = 3p2 + 1

Using a2 b2 = (a b)(a + b)

q(p2 2) = 3p2 + 1

Sub p = 4 into q =
q =

(b) Time taken to cut the lawn with old mower =


(c)

q =

.
.

q =
q = 3.5

= 3

= 3

3(5x)
15x
12x
x

=
=
=
=
=

3
7 + 3x
7 + 3x
7

(d)

= 10
= 10

10x(x 12.5)
10x2 125
2
10x 125x 4125
2x2 25x 825

= 3

= 10
.

(b)

17. (a) Time taken to cut the lawn with new mower =

q =

= 10
=
=
=
=

4125
4125
0
0 (shown)

2x2 25x 825 = 0


(2x 55)(x + 15) = 0
2x 55 = 0
or x + 15 = 0
2x = 55
x = 15 (rejected)
x = 27.5
Sub x = 27.5 into ,
Time taken to cut the lawn with new mower =

= 12 minutes

5/8

18. (a) Sub c = 5 and m = 4 into y = mx + c

Equation of the line


y = mx + c
where m is the gradient
and c is the y-intercept.

Equation of the line


y = 4x + (5)
y = 4x 5
(b) Sub y = h + 10 and x = h into y = 6x 5,
h + 10
h + 10
15
h

=
=
=
=

6(h) 5
6h 5
5h
3

= 4.22
= 221.76 cm3
10 litres = 10 000 cm3
Number of conical cups =

2 pupils
2 pupils
1 pupil
3 pupils
1 pupil
1 pupil

(b) The scores arranged in order of increasing magnitude are:


3, 3, 4, 4, 5, 6, 6, 6, 7, 9
Since we have an
Medium mark = mean of 5th and 6th values
even number of
=

scores, the median is
= 5.5 marks
the mean of the two


middle numbers
(c) Mean =

(c) When the line meets the y-axis, x = 0.


Coordinates of T = (0, 9)
19. (a) Volume of 1conical cup =

20. (a) Number of pupils who scored 3 marks


Number of pupils who scored 4 marks
Number of pupils who scored 5 marks
Number of pupils who scored 6 marks
Number of pupils who scored 7 marks
Number of pupils who scored 9 marks
Modal mark is 6.

12

1 l = 1000 ml
= 1000 cm3

45
(b)

Volume of orange juices in 45 cups =


=
Volume of orange juices left in container =
=
3
(c) Volume of hemisphere =

= 5.3
The mean is 5.3 marks.

45 221.76
9979.2 cm3
10000 9979.6
20.8 cm3

10 000 = 3
4772.727 = 3
r = 16.836 cm
16.84 cm (2 d.p.)

6/8

(d)

21. (a)
x
y

2
0

1
6

0
10

1
12

(b)

2
12

3
10

4
6

5
0

6
8

x2 + 3x
x2 + 3x + 6
2
x + 3x + 10 4
x2 + 3x + 6

=
=
=
=

6
0
y4
y4

Draw y = 4 on the graph,


x = 1.4 or x = 4.4

(c) Maximum point is (1.5, 12.25)

7/8

22. (a)
Distance (in km)

No. of times

11 d 30

31 d 50

51 d 70

71 d 90

(b) (i) Modal class 11 d 30


(ii) Median distance = mean of 15th and 16th values

=
= 47.5 km
(iii) Mean =
=

= 49.46
49.5 (3 sig. fig)
(c) (i) P(less than 35 km) =
=
(ii) P(between 45 km and 65 km) =
=
(iii) P(at least 85 km) =
=

8/8

SECTION A (50 MARKS)


1.

(a) Expand and simplify (3x y)2 4(2x + 3y)(y x).

[2]

(b) Simplify 2a 3b 2 1 a 2 .
4

[3]

2.

MATHEMATICS
(Secondary Two)
3.

End-Of-Year Examination 2014

(a) 8x2 2

[2]

(b) 3yz 6xy 5xz + 10x2

[2]

(a) Express x in terms of a, b and c, given that


(b) Simplify

4.

The number of marks is given in brackets [ ] at the end of each


question or part question.
The total number of marks for this paper is 100.
The use of an electronic calculator is expected, where appropriate.
Omission of essential working will result in loss of marks.
You are reminded of the need for clear presentation in your
answers.

(b)

5.

2
3 .

3x 1 2 6 x

xa
3c .
x b

[2]
[3]

Solve the following equations

2x 5
x
3

[2]

40 40

1
x x2

[3]

(a) 3

Answer all questions in Section A AND Section B.


If the degree of accuracy is not specified in the question, and if the
answer is not exact, give the answer to three significant figures.
Give answers in degrees to one decimal place.

Factorise the following

(2 hours)

IV

Solve the simultaneous equations.

[3]

2 x 3 y 13
3
1
x y 0
4
2

7 printed pages, including this cover page and answers sheet

1/7

6.

7.

A biased die is thrown. If the probability of getting a six is 3 times higher than
getting any one of the other five numbers, what is the probability of getting a
four?
[2]
In the diagram shown, AB = BC = CE and BE = ED. Prove that BAE is
congruent to ECD. Show your working clearly and state the case of
congruency.

9.

The figure below shows three sectors with their radii forming a regular hexagon
ABCDEF. The side of the hexagon is 12 cm. Find
(a) reflex angle ABC

[2]

(b) the total area of the shaded parts, leaving your answer in terms of .

[2]

[3]

A
8.

The diagram shows part of the curve y = 2(x + 1)(x 3). It cuts the x-axis at point
P and the y-axis at point Q. Write down the coordinates of
(a) P

[1]

(b) Q

[1]

10. An isosceles triangle ABC is inscribed in a circle with centre O.


If AB = AC = 13cm and BC = 10 cm, find the radius, r cm, of the circle.

[4]

y
x

13 cm
r cm
r cm O

y = 2(x + 1)(x 3)

10 cm

2/7

11. The distribution table below gives the ages of a group of 600 people:
Age (in completed years)
20 29
30 39
40 49
50 59
60 69
70 79
80 89

Number of people
56
87
165
184
73
23
12

(a) Complete the table.


(b) Write down the modal class of the distribution.
(c) Calculate an estimate mean age of the distribution.

Mid-Value

[1]
[1]
[3]

12. (a) For the Venn diagram below, express the shaded region in set notation. [1]

(b) In the Venn diagram shown below,


= {x : 4 x 20, x }
A = x : x is a multiple of 3
B = x : x is an odd number

(i) Add the set C to the Venn diagram where


C = {x : x is a prime number}

[1]

(ii) Find n(A B)

[1]

(iii) Find (A B) C

[1]

13. Study the following sequence of squares.


Row

Sum of digits of D

42

16

342

1 156

13

111 556

19

334

3 3342

(a) Write down the value of p and of q

[1]

(b) Find the sum of the digits in the answer to 333 333 3342.

[1]

(c) Express the sum of the digits of D for Row n in terms of n.

[2]

3/7

SECTION B (50 MARKS)


14. Mr Rich invested S$ 150 000 in Malaysia at 6 % per annum simple interest when
1
the exchange rate was S$ 1 = RM 2.25. After 5 years, he withdrew the interest
2
gained from his investment. The exchange rate then was S$ 1 = RM 2.06.

16. The diagram shows a solid made by joining together a pyramid and a cuboid.
The cuboid has a square base of sides 10 cm and height 30 cm. The perpendicular
height of the pyramid is 12 cm. Calculate

Calculate the amount in Singapore dollars, correct to the nearest cent, that he
would receive.
[4]

12

15. (a) Some children were asked how many lollipops they ate the previous day.
The table shows the results.
Number of lollipops ate
Number of children

0
7

1
9

2
3

30

3
x

(i) Write down the largest possible value of x given that the mode is 1.
[1]
(ii) Write down the largest possible value of x given that the median is 1.
[1]
(iii) Calculate the value of x given that the mean is 1.
[2]
(b) The mean of nine numbers is 15. If one of the numbers is removed, the mean
becomes 16. What is the number that was removed?
[3]

10

10
(a) the volume of the solid
(b) the height of each triangular surface of the pyramid, and
(c) the total surface area of the solid

[2]
[2]
[2]

130 of these solids are made. The faces of all the solids need to be painted.
The paint used is sold in tins, each containing enough paint to cover 0.25 m2.
How many tins are needed?

[3]

4/7

17. The following are the heights (cm) obtained by a class of 30 during a health
check.
155

156

157

160

162

165

155

158

162

162

151

169

170

166

161

153

168

163

164

158

160

171

170

165

159

156

157

161

163

167

18. The members of a gardening club decided to spend $36 on buying garden tools
and they found that a rake and a trowel together cost $7. If they spent all the
money on rakes alone, they could buy 3 more tools than they could if they spent
all the money on trowels.
(a) By taking the cost of a trowel as $q, write down expressions in terms of q for
(i) the cost of a rake,
[1]
(ii) the number of rakes which could be bought for $36.
[1]
(b) Write down an equation which q must satisfy, and show that it reduces to
q2 + 17q 84 = 0. Solve this equation and state the cost of a rake.
[5]

(a) Some of the heights are represented in a stem-and-leaf diagram below.


Complete the stem-and-leaf diagram for this data.
[2]
15

1 3

16

17

(b) What percentage of the students are taller than 1.58 cm?

[2]

(c) What is the mean height in cm?


(Correct your answer to 3 significant figures.)

[2]

(d) What is the mode of this data?

[2]

(e) What is the median of this data?


(Correct to 1 decimal place.)

[2]

5/7

20. Two unbiased dice are thrown. The possibility diagram when the two dice are
thrown is shown below. For example, (1, 3) represents that the first die shows a
number 1 and the second die shows a number 3.

19. Answer the whole of this question on a sheet of graph paper.

(a) Complete the possibility diagram.

6
5
4
3
2
1

ABCD is a square of side 8 cm and AEFG is a rectangle. If AE = GD = x cm,


show that the area of the rectangle AEFG is (8x x2) cm2.
[1]
Given that y = 8x x2 and some corresponding values are given in the following
table:
x
y

0
0

1
7

2
12

3
15

4
16

5
a

6
12

7
b

(a) Calculate the values of a and of b.

(1, 6)
(1, 5)

[1]

(3, 6)
(2, 5)
(2, 4)

(1, 3)

(5, 6)
(5, 5)

(3, 4)
(3, 3)

(4, 4)
(4, 3)

(3, 1)
3

(4, 1)
4

(2, 2)
(1, 1)
1

(6, 6)
(6, 4)

(5, 3)
(5, 2)
5

(6, 1)
6

(b) List all the possible outcomes that both the numbers are prime numbers and
find the probability that both the numbers are prime numbers.
[2]
(c) List all the possible outcomes that the difference of the two numbers is one
and find the probability that the difference of the two number is one. [2]

8
0
[1]

(b) Using a scale of 2 cm to 1 unit on the x-axis and 1 cm to 1 unit on the y-axis,
draw the graph of y = 8x x2 for values of x in the range 0 x 8.
[3]
(c) Write down the equation of the axis of symmetry of the graph.

[1]

(d) Find the maximum area of the rectangle AEFG.

[1]

(e) Use your graph to estimate the two values of x for which the area of the
rectangle AEFG is 8 cm2.
[1]

6/7

ANSWERS (dont peek till youve tried the question!)


1.

(bi)

(a) 17x2 2xy 11y2

2.

(b) 2a11b6

(a) 2(2x + 1)(2x 1)


(b) (z 2x)(3y 5x)

3.

4.
5.
6.

7
(a) x a 3bc (b)
3c 1
2 3x 1
(a) x

4
5

(b) x = 10 or x = 8

13. (a) 11115556; 25(b) 55

(aii) 12

(aiii) 2

(b) 13cm

(c) 1560 cm2, 82 tins

(b) 7

1
8

16. (a) 3400 cm2


17. (a)

7.

BAE ECD (SAS)

8.

(a) (1, 0)

(b) (0, 6)

9.

(a) 240

(b) 288 cm2

10. 7.04 cm
12. (a) A B

(c) 6n + 1

14. S$54065.53
15. (ai) 8

x = 2, y = 3

11. (b) 50 59

(bii)

(bii) 6

(c) 48.6

15

1 3 5 5 6 6 7 7 8 8 9

16

0 0 1 1 2 2 2 3 3 4 5 5 6 7 8 9

17

0 0 1

(b) 66 2 %
3
18. (ai) $(7 q)

(c) 161

(d) 162

(aii) 36
7q

(b) q = 21 or 4; $3

19. (a) a = 15, b = 7 (c) x = 4


6.85
20. (b) 1
4

(c)

(d) 16 cm2

(e) 161.5

(e) 1.15 or

5
18

7/7

SECTION A
1.

(a) (3x y)2 4(2x + 3y)(y x)


2 3
4 2
= 3
= (9x2 6xy + y2) 4(xy 2x2 + 3y2)
= 9x2 6xy + y2 + 4xy + 8x2 12y2
= 17x2 2xy 11y2
(b) (2a3b2)3

= (8a9b6)

a2

= (8a11b6)
= 2a11b6
2.

IV
Worked Solutions

= (8a9b6) a2

MATHEMATICS
(Secondary Two)

End-Of-Year Examination 2014

(a) 8x2 2 = 2(4x2 1)


1
=2 2
= 2(2x + 1)(2x 1)

Using a2 b2 = (a b)(a + b)

(b) 3yz 6xy 5xz + 10x2 = 3y(z 2x) 5x(z 2x)


= (3y 5x)(z 2x)
3.

(a)
3c(x b)
3cx 3bc
3cx x
x(3c 1)
x

(Dont peek till youve tried the question!)

(b)

= 3c
=
=
= a + 3bc
= a + 3bc

=

=
=

7 printed pages

1/7

4.

(a)

= x

5.

= x

2x 3y = 13
= 0
=

= x

x =

= x

y = 13
y = 3
Sub y = 3 into (2),
x =
x = 2

= 1
= 1
= 1

80 =
2
80 = x2 + 2x
0 = x2 + 2x 80
(x 8)(x + 10) = 0
x8 = 0
or
x + 10 = 0
x = 8
x = 10

(2)

3y = 13

= 1

x =
Sub (2) into (1),
2
3y = 13

4 2x = 3x
4 = 5x
x =
(b)

(1)

6.

Let P(getting 1) 1 unit


P(getting 2) 1 unit
P(getting 3) 1 unit
P(getting 4) 1 unit
P(getting 5) 1 unit
P(getting 6) 3 unit
Total probability = 8 units
8 units 1
1 unit

The probabilities of all the


outcomes must add to 1

Probability of getting a four =


7.

BA
BE
EBC
BAE

=
=
=
=

EC (given)
ED (given)
BEC BC = EC (Isos. )
ECD (SAS)

2/7

8.

(a) Substitute y = 0 into y = 2(x +1)(x 3),


0 = 2(x +1)(x 3)
0 = (x +1)(x 3)
x+1=0
or
x 3=0
x = 1 x = 3 (rej.)
Coordinates of P are (1, 0)

When the curve cuts


at x-axis, let y = 0.
When the curve cuts
at y-axis, let x = 0.

(b) Substitute x = 0 into y = 2(x +1)(x 3),


y = 2(0 +1)(0 3)
y = 2(1)(3)
y = 6
Coordinates of P are (0, 6)
9.

(a)

Each interior angle =

Each interior angle


in n-sided polygon

10. Let the mid-point of BC be M.


AM2 + BM2 =
52 + BM2 =
25 + BM2 =
BM2 =
BM =
BM2 + OM2
52 + (12 r)
25 + [122 2(12)(r) + r2]
25 + (144 24r + r2)
169 24r
169
r

Age (in completed years)

Number of people
(f)
56
87
165
184
73
23
12

20 29
30 39
40 49
50 59
60 69
70 79
80 89

= 12
= 144
Area of total shaded parts = 3 144
= 2 144
= 288 cm2

OB2
r2
r2
Mid-value of a class =
r2
0
24r
7.04 cm (3 sig. fig.)

=
=
=
=
=
=

11. (a)

ABC = 120
Reflex angle ABC = 360 120
= 240
(b) Area of major sector ABC =

AB2
132
169
144
12 cm

(b) Modal class of distribution 50 59


(c) Estimate mean age =
=

Mid-Value
(x)
24.5
34.5
44.5
54.5
64.5
74.5
84.5

Modal class is the class which


has the highest frequency

=
48.6 (3 sig. fig)

3/7

SECTION B

12. (a) A B
(b) (i)

14. S$150000 2.25 = RM 337500


Interest earned in 5.5 years =
.

=
= RM 111375


.
S$54065.5
Amount that he would receive = $54065.50

RM 111375 =
B = {5, 7, 9, 11, 13, 15, 17, 19}
(ii) A = {6, 9, 12, 15, 18}
(A B) = {5, 6, 7, 9, 11, 12, 13, 15, 17, 18, 19}
(A B) = {4, 8, 10, 14, 16, 20}
n(A B) = 6
(iii) (A B) C x is a multiple of 3, odd number and prime number
(A B) C =
3 is a prime number, but
multiples of 3 are not prime
13. (a) q 19 + 6
numbers.
= 25
p 11115556
p = 11115556 and q = 25
(b) Row number 9th
91=8
8 6 = 48
Sum of digits of D = 48 + 7 = 55
Sum of the digits to 333 333 3342 = 55
(c) (n 1) 6 = 6(n 1)
6(n 1) + 7 = 6n 6 + 7
= 6n + 1
Sum of the digits of D for Row n = 6n + 1

15. (a) (i) x 9


Largest possible value of x = 8
(ii) 7 + 9 = 16
16 3 = 13
x 13
Largest possible value of x = 12
(iii)

Mean =
1
0 + 9 + 6 + 3x
15 + 3x
2x
x

=
=
=
=
=

19 + x
19 + x
4
2

(b) Let the number be y.


16 8 + y = 9 15
128 + y = 135
y = 135 128
= 7
The number that was removed is 7.

4/7

16. (a) Volume of Pyramid =


=
=
Volume of Cuboid =
=
=
Volume of Solid =
=

base area height


102 12
400 cm3
Length Width Height
10 10 30
1000 cm3
300 + 400
3400 cm3

(b) Using Pythagoras theorem,


122 + 52 = h2
144 + 25 = h2
h = 13
Height of each triangular surface = 13 cm
(c) Area of 4 triangular surface of the pyramid
= 4 10 13
= 260 cm2
Total surface area = 260 + 10 30 4 + 10 10
= 1560 cm2
(d) Total surface area for the 130 solids
= 130 1560
1 m2 = 10000 cm2
2
= 202800 cm
= 20.28 m2
.
Number of tins needed for painting =
.
= 81.12
82

17. (a)
15

1 35 5 6 6 7 7 8 8 9

16

0011222334556789

17

001

(b) Number of students taller than 1.58 cm = 20


100%

% of the students are taller than 1.58 cm =

=
(c) Mean height


=

Mode
= the height that occurs most often

= 161.46
161 (3 sig. fig)
(d) Mode = 162 cm

(e) Medium mark = mean of 15th and 16th values


=

= 161.5 (1 d.p.)

Since we have an even


number of scores, the median
is the mean of the two middle
numbers

5/7

18. (a) (i) Let the cost of one rake be x and the cost of one trowel be q.
x+q=7
x=7q
Cost of one rake = $(7 q)
(ii) Number of rakes that can be bought with $36 =
(b) Number of trowels that could be bought with $36 =
= 3+
=
36q
36q
36q
3q2 + 51q 252
3(q2 + 17q 84)
q2 + 17q 84
(q 4)(q + 21)
q4 = 0
q = 4

or

=
=
=
=
=
=
=

(7 q)(3q + 36)
21q + 252 3q2 36q
15q + 252 3q2
0
0
0 (Shown)
0

19. Area of rectangle =


=
=
=

Length Breadth
AG AE
(8 x) (x)
8x x2 (shown)

(a) Sub x = 5 into y = 8x x2,


y = 8(5) (5)2
y = 40 25
y = 15
a = 15
Sub x = 7 into y = 8x x2,
y = [8(7) (7)2]
y = (56 49)
y=7
b=7

q + 21 = 0
q = 21 (rejected)

Substitute q = 4 into x = 7 q,
x = 7 (4)
x = 3
Cost of rake = $3

6/7

20. (a)

(b)
x
y

0
0

1
7

2
12

3
15

4
16

5
15

6
12

7
7

8
0

6
5
4
3
2
1

(1, 6)
(1, 5)
(1, 4)
(1, 3)
(1, 2)
(1, 1)
1

(2, 6)
(2, 5)
(2, 4)
(2, 3)
(2, 2)
(2, 1)
2

(3, 6)
(3, 5)
(3, 4)
(3, 3)
(3, 2)
(3, 1)
3

(5, 6)
(5, 5)
(5, 4)
(5, 3)
(5, 2)
(5, 1)
5

(4, 6)
(4, 5)
(4, 4)
(4, 3)
(4, 2)
(4, 1)
4

(6, 6)
(6, 5)
(6, 4)
(6, 3)
(6, 2)
(6, 1)
6

(b) All possible outcomes that the numbers are prime numbers
(2, 2), (2, 3), (2, 5), (3, 2), (3, 3), (3, 5), (5, 2), (5, 3), (5, 5)
P(both the numbers are prime numbers) =
1 is not a prime
number.

(c) All the possible outcomes that the difference of the two numbers is one
(1, 2), (2, 1), (2, 3), (3, 2), (3, 4), (4, 3), (4, 5), (5, 4), (5, 6), (6, 5)
P( difference of the two numbers is one) =
=

(c) x = 4
(d) Maximum area of rectangle AEFG = 16
(e) When y = 8, x = 1.15 or 6.85

7/7

SECTION A (50 MARKS)


1.

Express 0.0349848 as a decimal


(a) correct to 3 decimal places,
(b) correct to 4 significant figures.

2.

MATHEMATICS
(Secondary Two)

Evaluate
5 1
1
(a)
4
6 6
2
1
1 1

(b)
2 3
5
5 4

[1]
[1]

[1]
[1]

End-Of-Year Examination 2014


(2 hours)

3.

(a) (x 3)2 = 3x 11
8
(b)
x2
x 5

V
Answer all questions in Section A AND Section B.
If the degree of accuracy is not specified in the question, and if the
answer is not exact, give the answer to three significant figures.
Give answers in degrees to one decimal place.
The number of marks is given in brackets [ ] at the end of each
question or part question.
The total number of marks for this paper is 100.
The use of an electronic calculator is expected, where appropriate.
Omission of essential working will result in loss of marks.
You are reminded of the need for clear presentation in your
answers.

6 printed pages, including this cover page and answers sheet

Solve the equations

4.

If y

[2]

3x 5
1 5 z , express
and y
15
15

(a) x in terms of y
(b) x in terms of z

5.

[2]

[2]
[2]

(a) Calculate the value of k if the line x 2y = 10 passes through the point
(k, 3k).
[2]
(b) Without the use of a calculator, evaluate
1.3582 0.3582

[1]

1/6

6.

The following table shows the distribution of number of children per family in a
sample of 30 households.
No of children in a household
No of households

0
5

1
6

2
4

3
10

4
3

5
1

10. If = x 9, x , A = {multiple of 3}, B = {prime numbers}


Find
(a) C such that C A and C B.
(b) Set D such that D = A B.
(c) Set E such that E = A B

6
1

[1]
[1]
[1]

Find:
(a) the mean
(b) the median
(c) the mode

7.

Given that

ab
b
a b
(b)
a

a b
, evaluate
2 3

(a)

8.

( a b) 2 ( a b) 2

a
a
2
2
x .
(b)
x 1

2 2
(a) Find the size of an interior angle of a regular octagon.

11. A solid is formed by fixing 6 identical square-based pyramids exactly onto the 6
faces of a cube. If the length of a side of the cube is 3 cm and the volume of the
[3]
solid formed is 117 cm3, find the height of each pyramid.
12. (a) A model of an exhibition hall is made using a scale of 1 : 500. The volume
of the exhibition hall is 12 000 m3. Calculate the volume of the model,
giving your answer in cubic centimetres.
[2]

[2]
(b) A map is drawn to a scale of 5 cm to 1 km.
[3]

Calculate
(i) the area on the map which represents an actual area of 8 km2
(ii) the scale of the map in the form 1 : n

Express, as a single fraction, in its simplest form


(a)

9.

[1]
[1]
[1]

[2]

[2]

[2]
[1]

13. Given that the number of days (d) and the number of workers (w) painting the
wall varies inversely. On an occasion where there are 8 workers, they took only 5
days to complete the paint job.
(a) Write an expression of d in terms w.
[2]
(b) Find out the number of workers needed to clean up the garden in 4 days.
[2]

[2]

(b) The size of each interior angle of a regular polygon is five times the size of
each exterior angle. How many sides does the polygon have?
[2]

2/6

14. In the figure, ABC = CDE.

SECTION B (50 MARKS)

15. In a health survey, thirty old people were asked how many times they had visited
a doctor last year. The number of visits was given below:

1
5
3

18 cm

8 cm

0
1
0

0
2
1

2
0
1

3
2
2

1
0
3

1
1
2

0
3
1

2
2
0

4
4
1

6 cm
(a) Complete the frequency table below:

4 cm

No of visits
Frequency

Find the lengths of


(a) BE
(b) AD

[3]
[1]

1
9

[1]
4

5
1

(b) Draw a histogram to represent the above.

[2]

16. (a) (i) Simplify the following algebraic fractions:

18a 4 b 4 6a 2 b 3

7 x 2 y 3 14 xy 2
[2]

(ii) Make b the subject of the formula below:


b2 3
[2]
a
8
(b) A cyclist is traveling from Town A to Town C. He travels for 50 km, at a
constant speed of x km/h, until he reaches the point B, where his bicycle
chain breaks. He then walks the remaining 6 km from B to C at a constant
speed of (x 16) km/h.
Given that the total time for the whole journey from A to C is 4 hours, write
down an equation in x and solve to obtain his cycling speed.
[4]

3/6

17. A, B and C are vertices of a triangle. Given that A(4, 0), B(0, 3), C lies on the xaxis, and the equation of BC is 3y + 4x = 9, find
(a) the equation of AB in the form of y = mx +c
(b) the coordinates of C
(c) the area of triangle ABC

[2]
[2]
[2]

18. (a) In a survey, every member of a group of women was asked how many
children she had. The table below illustrated the results of this survey.
Find the mean, median and mode.
[3]

20. The length of the parallel sides of a trapezium are (x + 3) cm and (x + 9) cm


[4]
and its height is (3x 4) cm. If its area is 80 cm2, find the value of x.

21. (a) In the right-angled triangle ABC, D is a point on BC. Given that
AD = 15 cm, AC = 20 cm, BD = (x + 1) cm and DC = (x 1) cm, find the
value of
(i) x
(ii) AB

[3]
[2]

A
No of children per woman
No of women

0
5

1
6

2
8

3
3

4
1

20
15

(b) Another woman was surveyed and she had x number of children. Find
(i) the maximum value for x if the median is now changed to 1.5.
[1]
(ii) the value of x if the mean is changed to 1.625.
[1]
B
19. Using 1 cm to 1 unit for x-axis and 2cm for 5 units for the y-axis, draw the graph
of the equation y = 3x2 7x + 1 for 1 x 4.
x
y

1
11

1
3

3
7

4
21

(a) Hence solve the equation 3x2 7x + 1 = 0

[4]

(b) From the above graph, solve for 3x2 7x 4 = 0

[2]

x +1

x 1

(b) The diagram shows a conical glass of height 16cm. The diameter of its rim is
7cm. Some spherical jellies with radius 1.2cm are to be added until the liquid
in the glass reaches its brim. If there is already 86.6cm2 of liquid in the glass,
22
) [4]
find the least number of jellies needed to fill the glass. (Take =
7
7 cm

16 cm

4/6

22. Given that A = x : 15 < x < 26 and x is an integer.


(a) Write down the list of all elements in the set A

[1]

(b) If a number is chosen at random from A, find the probability that


(i) the number is a prime number
(ii) the number is an odd number

[1]
[1]

23. (a) A cylindrical tank with a hollow hemispherical base of radius 21cm contains
40 litres of water. What is the height of the water level in the tank?
[2]
22
)
(Take =
7
(b) The water in the tank was then drained through a valve at a rate of 1.8 litres
per minute till 6 hemispherical containers at internal diameter 28 cm are
exactly filled.
(i) Find the time needed to exactly fill up the 6 hemispherical containers.
22
and leave your answer corrected to the nearest minute.
Take =
7
[2]
(ii) How many litres of water are left in the tank?
[2]

21cm

5/6

ANSWERS (dont peek till youve tried the question!)

2
16. (ai) 6a b
xy

(aii) b 8a 3

1.

(a) 0.035

(b) 0.03500

2.

(a) 1 7
12

(b) 1
4

3.

(a) 4 or 5

(b) 3 or 6

4.

(a) x 15 y 5 (b) x 6 5 z
3
3

5.

(a) x = 2

(b) 1.716

6.

(a) 2.23

(b) 2.5

7.

(a) 1 2
3

(b) 1
2

20. 4 or 8 2 (rejected)
3

8.

(a) 4b

(b) 4
x

21. (ai) x = 8 or 22 (rejected)


3

9.

(a) 135

(b) 12

(b) 16 spherical jellies

10. (a) 6 or 9

1
17. (a) y 3 x 3 (b) C: (2 ,0)
4
4

(bi) x = 1

(a) x = 0.2, 2.2


(b) x = 0.5 and 2.8

(c) E = 3

11. 6.5

13. (a) d 40
w

(b) w = 10

14. (a) 9.5

(b) 1

(bii) x = 4

19. 1, 1 (table)

(b) D = 2, 3, 5, 6, 7, 9
(bi) 200 cm2

(c) 2.625 cm2

18. (a) Mean = 1.52, median = 2, mode = 2

(c) 3

12. (a) 96 cm3

(b) x = 20, speed = 20 km/h

(bii)1 : 20 000

(aii) AB = 12 cm

22. (a) A = {16, 17, 18, 19, 20, 21, 22, 23, 24, 25}
(bi) 3
10
23. (a) 42.9 cm

(bii) 1
2
(bi) 19 min

(bii) 5.50 litres

15. (a)
No of visits
Frequency

0
7

1
9

2
7

3
4

4
2

5
1

6/6

SECTION A
1.

(a) 0.0349848 0.035 (3 d.p.)


(b) 0.0349848 0.03500 (4 sig. fig.)

2.

(a)

4 =

MATHEMATICS
(Secondary Two)

End-Of-Year Examination 2014

V
Worked Solutions
(Dont peek till youve tried the question!)

(b)

2 3

=
=

3
3

=
=
=

8 printed pages

1/8

3.

(x 3)2
x 2(x)(3) + 32
x2 6x + 9
2
x 9x + 20
(x 4)(x 5)

(a)

=
=
=
=
=

x4 = 0
x = 4

or

(x 5)(x + 2) =
x + 2x 5x 10 =
x2 3x 18 =
(x + 3)(x 6) =
x+3 = 0
x = 3
2

(a)

(a) Mean =

=
2.23 (3 sig. fig.)

x6 = 0
x = 6
7.

(b) Medium mark = mean of 15th and 16th values


=

= 2.5
Since we have an even number
of scores, the median is the
(c) Mode = 3
mean of the two middle
(a)
=
numbers.
3a = 2b
=

=
=
=
=

10
10
10
2

+1

+1

(a) Sub x = k and y = 3k into x 2y = 10,


k 2(3k)
k 6k
5k
k

=
3x 5 = 1 5z
3x = 1 5z + 5
x =

5.

6.

x5 = 0
x = 5

y =
3x 5 = 15y
3x = 15y + 5
x =

(b)

8
8
0
0
or

Using a2 b2 = (a + b)(a b)

(b) 1.3582 0.3582


= (1.358 + 0.358)(1.358 0.358)
= 1.716 1
= 1.716

= x+2

(b)

4.

3x 11
3x 11
3x 11
0
0

(b)

=
=1
=1
=1
=

2/8

8.

(a)

10. A = {3, 6, 9}
B = {2, 3, 5, 7}

(a) C = 6 or 9

=
= 4b
(b)

(b) D = AB
= {2, 3, 5, 6, 7, 9}
(c) E = AB
= {3}
11.

=
=
9.

Volume of Pyramid =

(a) Each interior angles in a regular octagon


=
= 135

Volume of Pyramid = 117 6


= 19.5 cm3

(b) Let x be exterior angle,


Interior angle + exterior angle
5x + x
6x
x

Each interior angle


in n-sided polygon

19.5 = 3 3 h
19.5 = 3h
h = 6.5 cm
12. (a)

=
=
=
=

30 =
30n = 360
n = 12
The polygon has 12 sides.

1 cm
1 cm
(1 cm)3
1 cm3
96 cm3

:
:
:
:
:

500 cm
5m
(5 m)3
125 cm3
12000 cm3

5 cm
1cm
(1 cm)2
1 cm2
200 cm2

:
:
:
:
:

1 km
0.2 km
(0.2 km)2
0.04 km2
8 km2

180
180
180
30
(b) (i)

Each exterior angle =

base area height

(ii)

5 cm : 1 km
5 : 100000
1 : 20000

3/8

SECTION B

13. (a) d
d = , where k is the constant.

15. (a)

Sub d = 5 and w = 8 into d = ,

No of visits
Frequency

5 =
k = 40
d =

0
7

1
9

2
7

3
4

4
2

5
1

(b)

(b) Sub k = 40 and d = 40 into d = ,


4 =
w =
w = 10
10 workers are needed to clean up the garden in 4 days.
14. (a) ABC = CDE (given)

BCA = DCE (same angle)
ABC and EDC are similar.
=
=
BC = 13.5
BE = 13.5 4
= 9.5
(b)

=
=
AC = 9
AD = 9 8
= 1 cm

Ratios of corresponding
sides are equal

Ratios of corresponding
sides are equal

4/8

16. (a) (i)

(ii)

a
8a
8a 3
b

=
=

=
= b2 + 3
= b2
=

= 4
= 4
= 4
= 4
4x(x 16)
4x2 64x
2
4x 120x + 800
4(x2 30x + 200)
x2 30x + 200
(x 10)(x 20)

=
=
=
=
=
=

Equation of the line


y = mx + c
where m is the gradient
and c is the y-intercept.

=
y- intercept, c = 3
Equation of AB is y = x + 3

Coordinates of C are

Time taken from B to C

(b) Given that equation of BC


3y + 4x = 9
Sub y = 0 into equation of BC,
0 + 4x = 9
x =

(b) Time taken from A to B

17. (a) Gradient, m =

56x 800
56x 800
0
0
0
0

When the line lies


on x-axis, ycoordinate = 0

(c) Area of ABC = base height


= AC OB
= 4

= 3
=
= 2.625 units2

x 10 = 0
or
x 20 = 0
x = 10 (rejected)
x = 20
His cycling Speed = 20 km/h

5/8

18. (a) Mean =

19.

x
y

1
11

0
1

1
3

2
1

3
7

4
21

=
1.52 (3 sig. fig.)
th position

Middle position =
Median = 2

Mode = 2 children per woman


(b) (i) Median for 23 women = 2
Median for 24 woman = mean of the 12th and 13th number 1.5
x can be 0 or 1 if the median is 1.5
Maximum value of x = 1
(ii)

Mean =

1.625 =
1.625 =
35 + x = 1.625 24
x = 39 35
= 4

(a) When y = 0, x = 0.1 or x = 2.2


(b)

3x2 7x 4
3x2 7x 4 + 5
3x2 7x + 1
y

=
=
=
=

0
5
5
5

When y = 5, x = 0.6 or x = 2.8

6/8

20. (a)

Area of trapezium =
80 =
160
160
0
0
0
0

AB2 + 4(82)
AB2 + 4(64)
AB2 + 256
AB2
AB

(x + 3 + x + 9) (3x 4)

(2x + 12)(3x 4)
6x2 8x + 36x 48
6x2 + 28x 208
2(3x2 + 14x 104)
3x2 + 14x 104
(3x + 26)(x 4)
3x + 26 = 0
or
x4 = 0
3x = 26 (rejected)
x = 4
21. (a) (i)

(ii) Substitute x = 8 into AB2 + 4x2 = 400,

(sum of parallel lines) height

=
=
=
=
=
=

AB2 + BD2
2
AB + (x + 1)2
AB2 + [(x)2 + 2(x)(1) + (1)2]
AB2 + (x2 + 2x + 1)
AB2 + x2 + 2x

=
=
=
=
=

AD2
152
225
225
224

AB2 + BC2
AB2 +[(x + 1)+(x 1)]2
AB2 + (2x)2
AB2 + 4x2

=
=
=
=

AC2
202
400
400

(2) (1),
[AB2 + 4x2] [AB2 + x2 + 2x] = 400 224
3x2 2x = 176
2
3x 2x 176 = 0
(3x + 22)(x 8) = 0
3x + 22 = 0
or
x8 = 0
3x = 22 (rejected)
x = 8

(b)

Using Pythagoras
theorem

=
=
=
=
=

Volume of conical glass =

16
16

=
= 205.33 cm3
Volume of 1 spherical jelly =

(1)

400
400
400
144
12

(1.2)3

= 1.728
= 7.241 cm3
(2)

Volume of liquid needed to fill the glass = 205.33 86.6


= 118.73 cm3
Number of jellies needed to fill the glass = 118.73 7.241
16
22. (a) A = {16, 17, 18, 19, 20, 21, 22, 23, 24, 25}
(b) (i) P(the number is a prime number) =
(ii) P(the number is an odd number) =

7/8

23. (a)

Volume of hemisphere =

= 213
= 19404 cm3
Volume of cylinder +hemisphere =
40000 + 19404 =
21
441
59404 =
59404 = 1386h
h 42.9 cm (3 sig. fig.)
(b) (i)

Volume of hemisphere =
=

14

5749.33
Volume of 6 hemispheres = 5749.33 6
= 34495.98
1.8l = 1800 cm3
Time needed to fill the containers = 34495.98 1800
19 (nearest minutes)
(ii) Volume of water left = 40 000 34495.98
= 5504.02 cm3
5.50 l (3 sig. fig)
1 l = 1000cm3

8/8

SECTION A (50 MARKS)


1.

Express, correct to 2 significant figures


(a) 237.135
(b) 0.0005193

2.

MATHEMATICS
(Secondary Two)

[1]
[1]

In the diagram shown below, ADE is the enlargement of ABC with scale factor
2.
(a) Where is the centre of enlargement?
(b) Find x and express y in terms of n or/and m.
B

End-Of-Year Examination 2014

C
m cm

12 cm

y cm

VI

16 cm

Answer all questions in Section A AND Section B.


If the degree of accuracy is not specified in the question, and if the
answer is not exact, give the answer to three significant figures.
Give answers in degrees to one decimal place.

7 printed pages, including this cover page and answers sheet

x cm

n cm

(2 hours)

The number of marks is given in brackets [ ] at the end of each


question or part question.
The total number of marks for this paper is 100.
The use of an electronic calculator is expected, where appropriate.
Omission of essential working will result in loss of marks.
You are reminded of the need for clear presentation in your
answers.

[2]
[4]

D
3.

A camp was planned for a group of 20 girls and food was bought to last for 15
days. If the number of pupils attending the camp was increased to 25, how long
would you expect the same amount of food to last?
[2]

4.

Express

5.

Factorise completely 36y2 (x2 +2xy +y2).

7
h 14

as a single fraction in its simplest term.


12 h 4 h 2

[2]
[3]

1/7

6.

11. The diagram below shows 2 triangles A and B drawn on a plane.

The table below shows the marks obtained by a group of 40 pupils in a test.
Marks
80 - 89
70 - 79
60 - 69
50 - 59

No of pupils
5
10
n
3

(a) Find n.

[2]

(b) If the minimum mark for a distinction is 70, calculate the percentage of
pupils who scored a distinction.
[2]

7.

Solve the following equation 7w = w2

[2]

8.

Given that a(b + 1) 3b = 5a, express b in terms of a.

[2]

B
9.

Solve the simultaneous equations


3x 4y = 2
2x + 7 = 6y

-4

-2

[3]

-2
10. If

1 , find d.

[2]

B is the image of A under a single transformation of u.


Describe the transformation u.

[2]

2/7

12. A triangular plot of land ABC is such that AB = 25 m, BC = 20 m and


AC = 15 m. A straight fence, p m long, is put up from the corner C to a point D
on edge AB. The fence is perpendicular to the edge AB and q m from A.

14. (a) The equation of a straight line is

1
y 2x 1.
4

Find the gradient and the y-intercept of the straight line.

[2]

(b) Draw the following graphs on the grids provided.

(i) y + 2x = 0

25
q

(ii) 2y = x + 4

[2]

20
p

15
C
(a) Using Pythagoras theorem, write down 2 equations involving p and q. [2]
(b) Hence, find the value of
q.

[2]

13. An oil trader bought some oil for $500. He paid $x for each litre of oil.
(a) Find, in terms of x, an expression for the number of litres of oil he bought.
[1]
(b) Due to leak, he lost 3 litres of oil. He sold the remainder of the oil for $1
litre more than he paid for it. Write down an expression, in terms of x, for
sum of money he received and show that it is equal to
( 500 3 x )( x 1)
.
[2]
$
x
(c) He made a profit of $17. Write down an equation in x and show that it
[3]
reduces to 3x2 + 20x 500 = 0.

(d)

Solve 3x2 + 20x 500 = 0.

[2]

3/7

15. The diagram shown below is a pictograph which shows the number of tickets
sold in 3 particular days.
No of tickets sold
Wednesday

[2]

(b) The hire purchase price of $860 is paid by a cash deposit of 22% and the
remainder by 12 equal monthly installments. Calculate

Friday

represents y tickets
If the mean number of tickets sold per day is 600, calculate
(a) the total number of tickets sold in 3
[2]

(b) the number of tickets sold on


Thursday.

16. The cash price of a television set is $800. The hire purchase price is $860.
(a) Calculate the percentage increase in price for hire purchase.

Thursday

days.

SECTION B (50 MARKS)

[2]

(i) the cash deposit

[1]

(ii) each monthly installment.

[2]

17. Two places P and Q are along a straight river and PQ = 6 km. A boat usually
travels from P to Q at a constant speed of x km/h.
(a) On Tuesday the weather was worse than usual, so the boat travelled
y km/h slower than its usual speed. The journey form P to Q that day took
48 minutes. Show that x y = 7.5
[2]
(b) On Wednesday the weather was better than usual, so the boat travelled
y km/h faster than its usual speed. The journey from P to Q that day took
36 minutes. Form and simplify another equation in x and y to represent
this information.
[2]
(c) Solve these two equations in x and y, and hence find the time that the boat
usually takes to go from P to Q.
[3]

4/7

18. (a) ABCDE is a pentagon with A = B = C = D = 2x, E = x and


AB = BC = CD.
(i) How many lines of symmetry does the pentagon ABCDE have?

[1]

(ii) Calculate the value of x.

[2]

20. (a) In the figure, X and Y are the centres of two overlapping circles. If area of
each circle is 5 cm2, what is the area of the rectangle ABCD? Give your
answer in terms of .
[4]

(b) A map is drawn to a scale of 1: 400 000.


(i) Find the distance in kilometres, between two schools which are
represented on the map by points 6.5 cm apart.

[1]

(ii) An island covers an area of 640 km2. Find, in square centimetres, the
area representing the island on the map.
[2]
19. Of the 40 students in a class, 25 like Geography and 20 like Literature. It is given
that
= Students in the class
G = Students who like Geography
L = Students who like Literature

(b) The given solid is made up of a cone and a right cylinder. The radius of the
cylinder is 4.5 cm and the height is 20 cm.
(i) If the slant height of the cone is 15 cm, find the perpendicular height of
the cone.
[2]
(ii) Find the volume of the solid. Round off your answer to 2 decimal
places.
[3]

(a) Express in set notation to illustrate the statement There are students who
like Geography but do not like Literature.
[1]
(b) Let n(G L) = x.
(i) Express n(G L) in terms of x.

[1]

(ii) Find the smallest possible value of x.

[1]

(iii) Find the largest possible value of x.

[1]

20 cm

4.5cm

5/7

21. A class of 27 children took a Mathematics test. Some of their scores are
represented in the stem and leaf diagram below.
0
1
2
3
4

23. Answer the whole of this question on a sheet of graph paper.


A building has vertical walls and stands on horizontal ground. A coin is thrown
from a window of the building. At any instance the horizontal distance of the
coin from the building is x metres and the vertical height of the coin above the
ground is y metres. Some corresponding values of x and y are given in the
following table.

7
8 4
0
5 9
2

x
y

The other scores are given below.


25
30

37
14

14
27

47
29

11
32

27
38

40
45

23
21

9
34

29
28

(a) Construct a stem and leaf diagram to represent the scores of all 27 children.
[3]
(b) For the whole class, find
(i) the modal score
(ii) the median score
(iii) the mean score

[1]
[1]
[1]

1
24

2
26

3
26

4
24

5
20

6
14

7
6

(a) Using a scale of 2 cm to represent 1 metre on the x-axis and 2 cm to


represent 5 metres on the y-axis, draw x and y axes for 0 x 8 and
0 y 30 Plot the points given in the table and join them with a smooth
curve.
[3]
(b) Write down the height of the window above the ground.

[1]

(c) From the graph estimate


(i) the greatest height above the ground reached by the coin,

[1]

(ii) the horizontal distance from the building of the point where the coin
strikes the ground.
[2]

22. A bag contains 5 green balls, 3 red balls, 2 white balls and 1 blue ball.

(iii) the horizontal distance travelled by the coin whilst it is more than 23
metres above the ground.
[1]

A ball is chosen at random. Find the probability that


(a) the ball is red
(b) the ball is not white
(c) the ball is green or blue

0
20

[1]
[1]
[1]

(d) A wall of height 7 metres is situated d metres from the building. Given that
the coin passes 4 metres above the top of the wall, use your graph to estimate
the value of d.
[2]

6/7

ANSWERS (dont peek till youve tried the question!)


1.

(a) 240

(b) 0.0052

2.

(a) A

(b) x = 8, y = 2n

3.

12 days

4.

2h 21
6h 2

5.

(5y x)(7y + x)

6.

(a) n = 22

7.

0 or 7

8.

9.

x = 4, y = 2.5

(bi) 26 km
19. (a) G L
(bii) 5

(bii) 40 cm2
(bi) 45 x
(bii) 20

20. (a) 30 cm2

(bi) 14.3 cm

(bii) 1575.59 cm3

21. (a)
0
1
2
3
4

(b) 37.5%

4a
a 3

(bi) 14

(bii) 28

(biii) 27.2

3
11

(b) 9
11

(c)

22. (a)

10. d 3 3
7

23. (b) 20 m

11. Enlargement with scale factor 2 at centre (4, 7)

79
14448
013577899
0245789
0257

(ciii) 3.5 m

(ci) 26.25 m

6
11

(cii) 7.65 m

(d) 6.4 m

12. (a) p2 + q2 = 152, p2 + (25 q)2 = 202


(b) q = 9
13. (a) 500
x

(c) (500 3x)( x 1) 500 17


x

(d) x = 10 0r 50
3

14. (a) gradient = 8, y-intercept = 4


15. (a) 1800

(b) 360

16. (a) 7.5%

(bi) $189.20

17. (b) x + y = 10

(c) x = 8.75, y = 1.25 , 0.686 h

18. (ai) 1

(aii) 60

(bii)$55.90

7/7

SECTION A
1.

(a) 237.135 240 (2 sig. fig.)


(b) 0.0005193 0.00052 (2 sig. fig.)

2.

(a) Centre of enlargement A


(b) ABC is similar to ADE under enlargement.
=
=
= 2 (scale factor)

MATHEMATICS
(Secondary Two)

= 2
= 2
x = 16 2
= 8

End-Of-Year Examination 2014


VI

= 2

y = 2n
3.

Worked Solutions
(Dont peek till youve tried the question!)

20 girls
10 girls
5 days
25 girls

15 days
30 days
60days
12 days

If the number of pupils increased to 25, the same amount of food will
last for 12 days.
4.

=
=
=

7 printed pages

1/7

5.

6.

36y2 (x2 + 2xy + y2)


= 36y2 (x + y)2
= (6y)2 (x + y)2
= (6y x y)(6y + x + y)
= (5y x)(7y + x)
(a) Total number of pupils
5 + 10 + n + 3
n
n

18y 21
10y
10y
y

Using a2 b2 = (a + b)(a b)

=
=
=
=

40
40
40 10 5 3
22

7.

7w
w 7w
w(w 7)
w
2

8.

a(b + 1) 3b
ab + a 3b
b(a 3)
b

9.

=
=
=
=

4 + 8y
4 + 21
25
2.5

Sub y = 2.5 into (2),


x =
= 4
10.

= 1
d = 1 3

(b) Number of students who scored 70 mark and above = 15


100
% who scored distinction =
= 37.5%

=
=
=
=

=
11. The diagram shows 2 similar A and B. B is an enlargement of A.
We can say the transformation u is an enlargement of scale factor 2 at centre
(4, 7).
Scale factor
12. (a)
DC2 + AD2 = AC2

=
(1)
p2 + q2 = 152

(2)
p2 + (25 q)2 = 202

w2
0
0
7 or 0

= 5a
= 5a
= 4a
=

p2 = 152 q2

(b)

(3)

Sub (3) into (2),

3x 4y = 2
2x + 7 = 6y
x =

(1)
(2)

152 q2 + (25 q)2


15 q2 + 252 50q + q2
252 50q + q2 q2
50q
q
2

=
=
=
=
=

202
202
202 152
252 202 + 152
9

Sub (2) into (1),


3

4y = 2
= 2 + 4y

2/7

13. (a)

$x 1 litre of oil
$1

14. (a)

litre of oil

$500

litre of oil

y = 1 + 2x
y = 4(1 + 2x)
y = 4 + 8x
Gradient = 8, y-intercept = 4

No. of litres he bought =

(b) (i) y + 2x = 0

(b) Selling price for 1 litre of oil = $ (x + 1)


3
Number of litre of oil sold =

3 (x + 1)

Sum of money he received =

x+1

=
= $

(shown)
2

(c) Total sum of money Cost = Profit


500 = 17

500 +

500 = 17

3x 3 500 = 17
3x 3 = 17
3x = 20
500 3x2 = 20x
3x 20x + 500 = 0 (shown)
2

(d)

y + 2x = 0

3x 20x + 500 = 0
(3x + 50)(x 10) = 0
3x + 50 = 0
3x = 50
x =

or

x 10 = 0
x = 10

3/7

15. (a) Total number of tickets sold = 3 600


= 1800
(b) 10 units 1800
1 unit 180
2 units 180 2
= 360
Number of tickets sold on Thursday = 360

(b) (ii) 2y = x + 4
y

3
2y = x + 4
2

4/7

SECTION B

(c) From (2),


x = 10 y
Sub (3) into (1),
10 y y = 7.5
10 2y = 7.5
2y = 2.5
y = 1.25
x = 10 1.25
= 8.75

16. (a) Increase in price in hire purchase = 860 800


= $60
% increase in hire purchase =
100
= 7.5 %
860

(b) (i) Cash deposit =

(3)

(ii) Monthly installment =

48 minutes =

hour

(1)

(b) On Wednesday:
Usual speed x km/h
New speed (x + y) km/h
Time taken =
=
36x + 36y = 360
x + y = 10

(ii) Sum of interior angles in pentagon = (5 2) 180


4(2x) + x = 540
Sum of interior angles
9x = 540
in n-sided polygon
x = 60
= (n 2) 180
(b) (i)
1 cm : 400000 cm
1 cm : 4 km
6.5 cm : 6.5 4 km
= 26 km
Distance between the two schools = 26 km

18. (a) (i) Number of lines of symmetry = 1

=
48x 48y = 360
x y = 7.5 (shown)

0.686 hour (3 sig. fig.)

= $55.90
17. (a) On Tuesday:
Usual speed x km/h
New speed (x y) km/h

Time taken =

Time the boat usually takes from P to Q =

= $189.20

(ii)
36 minutes =
(2)

hour

1 cm : 4 km
1 cm2 : 16 km2
cm2 : 640 km2
= 40 cm2
Area on the map = 40 cm2

5/7

19. (a) There are students who like Geography but do not like Literature
= L G
(b) (i) Given n(G L) = x

20. (a)

Area =
5 =
r2 =

r2
r2

r =

L
25 x x 20 x

Area of ABCD = AD AB
= 3r 2r
= 3

= 6
n(G L) = 25 x + x + 20 x
= 45 x
(ii) n(G L) is smallest when n(G L) = n( )
45 x = 40
x = 5
The smallest possible value of x is 2.
(iii) n(G L) is largest when L G
Then n(G L) = n(L)
x = 20
The largest possible value of x is 20.

cm2

(b) (i) Let l be the slant height and h be the perpendicular height.
r2 + h2 = l2
4.52 + h2 = 152
Using Pythagoras theorem
h2 = 15
4.5
h = 204.75
14.3 cm (3 sig. fig.)
(ii) Volume of solid = Volume of cylinder + Volume of cone
= r2h + r2h
= 4.52 20 +
= 1575.587
1575.59 (2 d.p.)

4.52 14.3

6/7

21. (a)

23. (a)
0
1
2
3
4

79
14448
013577899
0245789
0257

(b) (i) Modal score = 14


(ii) Middle position =
Median score = 28

th = 14th

(iii) Mean score


= 27.2
22. (a) P(the ball is red) =
(b) Number of the balls that are not white = 9
P(the ball is not white) =

(b) Height of window = 20 m


(c) (i)

(c) Number of the balls that are green or blue = 6


P(the ball is green or blue) =

Greatest height = 26.25 cm

(ii) Horizontal distance = 7.65 m


(iii)

Distance travelled = 4.25 0.7


= 3.5 m

(d) d = 6.4 m

7/7

SECTION A (50 MARKS)


1.

(a) Peter bought a digital camera for $577.50. This price included a Goods and
Services Tax (GST) of 5%. Calculate the GST that was paid.
[2]
(b) A group of 20 boys took 15 days to finish building a model aeroplane. If the
number of boys were to increase to 25, how long would they take to finish
building an identical model aeroplane?
[2]

MATHEMATICS
(Secondary Two)

2.

(a) Evaluate 2357 2 2356 2357

[1]

(b) Using algebraic rules, calculate 839752 83980 83970

[1]

End-Of-Year Examination 2014


(2 hours)

3.

VII
Answer all questions in Section A AND Section B.
If the degree of accuracy is not specified in the question, and if the
answer is not exact, give the answer to three significant figures.
Give answers in degrees to one decimal place.
The number of marks is given in brackets [ ] at the end of each
question or part question.
The total number of marks for this paper is 100.
The use of an electronic calculator is expected, where appropriate.
Omission of essential working will result in loss of marks.
You are reminded of the need for clear presentation in your
answers.

4.

(a) Factorize x y 49 y 2
3
9

(b) Expand and simplify y 2 y 3 y y 2 y 5 .

[2].
[2]

If y varies directly as (2x 3), and y = 21 when x = 5,


(a) express y in term of x

[2]

(b) find the value of y when x = 7

[1]

(c) find the value of x when y = 63

[1]

7 printed pages, including this cover page and answers sheet

1/7

5.

The figure shows a right-angled triangle PQR. Find the

8.

Score
Frequency

x 14

6.

(a) value(s) of
[3]
(b) perimeter of the triangle.

(a) Solve the equation 5 25


p
(b) Solve the simultaneous equations.
5x 4 y 0

9.

7.

2
6

(a) In the Venn diagram, shade A:

3
6

4
2

5
4

6
2
[1]

[2]
[1]

[1]

[1]
(b) In the Venn diagram, shade A B:
[3]

[1]

4 x 5 y 18
(c) Solve the equation x 1 x 2 2 x 3 .
3
5
10

1
8

(i) Write down the modal score.


(ii) After the 27th throw the median score was 2. What was the least
possible score on the 28th
throw?
[2]
1
(b) Given that v u at 2 , express a in terms of v , u and t.
2

x+4

(a) A six-faced die was thrown 28 times. The table shows the number of times
that each possible score occurred.

[3]
(c) In the Venn diagram, shade (A B):

[2]

A polygon has n sides. Three of its interior angles are 120, 130 and 140 while
the remaining (n 3) interior angles are each equal to 110. Find n.
[3]

2/7

12. The diagram shows a straight line AB with equation y 1 x 5 .


3
(a) Write down the coordinates of A.

[1]

(b) The line AB cuts the x-axis at the point B.


[1]
Write down the equation of the line passing through B parallel to y-axis.
10. There are 240 adults and 160 children at the cinema watching a movie. 5 of the
8
adults and 50% of the children are male. If one person left the cinema before the
movie ends, find the probability that the person is
(a)
(b)
(c)
(d)

an adult
a man
a girl
female

(c) Given that the line CD is parallel to AB and passes through the point
D (0 , 3), write down the equation of the line CD.

[2]

(d) Find the area of the parallelogram ABCD.

[2]

[1]
[1]
[1]
[1]

11. Given that ABC is similar to ADB, AD = 1 cm, AB = 2 cm and ABC = ADB
= 90, find the length of CD.
[3]

D (0,3)

D
1 cm

2 cm

3/7

SECTION B (50 MARKS)


6
3
13. (a) Express as a single fraction

2x 3 x 1

15. The table shows the number of cars owed by 25 families.

[2]

(b) Given that (a + b)2 = 256 and 6ab = 54, calculate the value of 4(a2 + b2).
[3]
(c) A salesman had a basic salary of $30 000 in 1999. In 2000, his basic salary
was increased to $34 000.
(i) Calculate the percentage increase in his basic salary from 1999 to 2000.
[1]
(ii) In 2000 he was paid a bonus of 2 % of the value of his sales. His total
income was $38 400. Calculate the value of the sales that he made that
year.
[1]

14. A small cinema has a capacity of 120 seats and is always full if the price of each
ticket is $8.
(a) Write down the revenue obtained.

2
0
2
1
3

0
1
3
2
4

3
1
6
0
1

4
2
1
3
2

1
3
0
2
1

(a) Complete the table below using the given information.


No of cars
No of families

[1]

(b) Find
(i) the mean number of cars
(ii) the median number of cars
(iii) the modal number of cars

[1]
[1]
[1]

16. The diagram below shows a circle with centre O. The diameter is 6.8 cm and the
arc length of AB is 4.7 cm. Take to be 3.14.

[1]
A

(b) For every $2 increase in the price of each ticket, the number of patrons
decreases by 5. Find the revenue when the price of each ticket is $12. [1]
O

(c) If x is the number of $2 increases in the price of tickets, show that the
[3]
revenue in dollars is given by 960 + 200x 10x2.
(d) Find the two values of x which give a revenue of $1 950.

[2]
(a) Calculate the angle AOB, correct to 1 decimal place.
(b) Calculate the area of the sector AOB, correct to the nearest cm2.

[2]
[2]

4/7

17. A cone is cut out of a hemisphere of diameter 45 cm. The base diameter of the
cone is found to be one-third that of the hemispheres.
45 cm

19. (a) Solve the equation x 2 x x 2


[2]
3
2
(b) 472 man-hours are needed to renovate an office. A renovation company
employs workers for 8 hours a day. Find the minimum number of workers
needed to complete the renovation in 6 days.
[3]
20. The universal set and the sets O, P and M are given by
= x : x is an integers such that 7 x 30,
O = x : x is an odd number,
P = x : x is a prime number,
M = x : x is a multiple of 5.

(a) Find the volume of the remaining solid, giving your answer correct to one
decimal place.
[3]
(b) Find the total surface area of the remaining solid, giving your answer correct
to 3 significant figures.
[4]
[Take = 3.14]
18. A box contains 6 black pens, 4 blue pens and 3 red pens. The pens are drawn at
random from the box, one after the other, and are not replaced. Expressing your
answer as a fraction, find the probability that
(a) the first pen drawn is blue

[1]

(b) the first and second pen are blue

[2]

(c) If the two blue pens drawn are placed back into the box, find the
probability that the next pen drawn will be black.

[1]

(a) Draw a Venn Diagram to illustrate the above sets.

[2]

(b) State the value of n(O M) and n(O M)

[2]

21. Answer the whole of this question on a sheet of graph paper.


The variables x and y are connected by the equation, y = x2 + x 6.
Some of the corresponding values of x and y are given in the table below.

x 4
y 6

3
a

2
4

1
6

0
6

(a) Calculate the value of a and of b.

1
b

2
0

3
6
[2]

(b) Using a scale of 2cm to represent 1 unit on the x-axis and 1cm to represent
[3]
1 unit on the y-axis, draw the graph of y = x2 + x 6 for 4 x 3.
(c) State the minimum value of y = x2 + x 6

[1]

(d) By drawing another line on your graph, find the solutions to the equation
[2]
2x2 + 2x 14 = 0

5/7

ANSWERS (dont peek till youve tried the question!)


1.

(a) $27.50

(b) 12 days

2.

(a) 2357

(b) 25

3.

(a)

4.

(a) y = 6x 9

(b) 33

5.

(a) x = 30

(b) 80 units

6.

(a) p = 1
5

(b) x = 8, y = 10

x 2 y x

7.

n=6

8.

(ai) 1

AA

8
y
3

(b) 2 y y 2 3

10. (a) 3
5

(b) a

2 v u
2

13. (a)

14. (a) $960


B

(c) 1
5

(d) 17
40

(b) x = 15

1
(c) y = x+3
3

(d) 30 units2

15
(2 x 3)( x 1)

(ci) 13 1 %
3

(a)

(b) 3
8

11. CD = 3 cm

(c) x = 6.5

12. (a) (0, 5)


(aii) 3

BB

(c) 12

t
9.

(c)

(cii) $220 000


(b) $1320

(bi) 1.92
16. (a) 79.2

(d) 9 or 11

15. (a)
No of cars
No of families

(b)

(b) 238

0
4

1
7

(bii)2

2
6

3
5

4
2

5
0

6
1

(biii)1

(b) 8 cm2

17. (a) 22519.7 cm3 (b) 5160 cm2


B

6/7

18. (a)

4
13

(c)

6
13

(b)

19. (a) x = 8

1
13

(b) least number of workers = 10

20. (a)

O
9, 21,
27

7, 11,
13,17,
19,23,
29

15,25
6,8,12,14
16,18,22,24
,26,28

10,20,
30

(b) n(O M) = 2 and n(O M) = 15


21. (a) a = 0, b = 4 (c) 6.25
(d) draw y = 1, x = 3.2, 2.2

7/7

SECTION A
1.

(b)

MATHEMATICS
(Secondary Two)
2.

End-Of-Year Examination 2014

Price of digital camera $577.50


105% $577.50
.
5
GST =
= $27.50

(a)

20 boys 15 days
1 boys (15 20) = 300 days
25 boys
= 12 days
The 25 boys will take 12 days to finish building the model aeroplane.

(a) 23572 2356 2357


= 2357(2357 2356)
= 2357

Factorise, do not use


calculator to evaluate.

(b) 839752 83980 83970


= 839752 (83975 + 5 83975 5)
= 839752 (839752 52)
= 839752 (839752 52)
= 25

VII
Worked Solutions
(Dont peek till youve tried the question!)
3.

(a)

Using a2 b2 = (a + b)(a b)

=
=
(b) y(2 y)(3 y) + y2(y + 5) = y(6 5y + y2) + y3 + 5y2
= 6y 5y2 + y3 + y3 + 5y2
= 2y3 + 6y
= 2y(y2 + 3)

7 printed pages

1/7

4.

(a)

y
21
k
y

=
=
=
=

(b)

k(2x 3)
k(10 3)
3
6x 9

If y is directly proportional to x,
then y = kx, where k is a constant
and k 0.

(b) When x = 7,
y = 6(7) 9
= 33

(a)

PQ2 + QR2
x + (x 14)2
x2 + x2 28x + 196
x2 36x + 180
(x 30)(x 6)
x 30 = 0
or
x = 30
2

(b)

6.

(a)

25p = 5
p =
p =

4x + 5y = 18

(2)

+ 5y = 18
1

=
=
=
=
=

=
y =
x =
=

Using Pythagoras theorem


PR2
2
(x + 4)
x2 + 8x + 16
0
0
x6 = 0
x = 6 (rejected)

(c)

18
10
10
8

=
=

10(5x + 5 3x + 6)
50x + 50 30x + 60
20x + 110
10x
x

Perimeter of = PQ + PR + QR
= 30 + 34 + (30 14)
= 80 units
= 25

(1)

Sub (1) into (2),


4( ) + 5y = 18

(c) When y = 63,


63 = 6x 9
6x = 63 + 9
x = 12
5.

5x + 4y = 0
5x = 4y
x =

7.

=
=
=
=
=

15(2x + 3)
30x + 45
30x + 45
65
6.5

Sum of interior angles in n-sided polygon


120 + 130 + 140 + (n 3)110
390 + 110n 330
60 + 110n
70n
n

=
=
=
=
=
=

(n 2) 180
(n 2) 180
180n 360
180n 360
60 + 360
6

2/7

8.

(a) (i) modal score = 1


(ii) Median = mean of 14th and 15th values
=
x can be 1 or 2 if the median is 2.

Modal score = the


score that occurs
most often

(c) (A B)

AA

BB

The least possible score is 1.


(b)

v = u+
= vu

10. (a) P(the person is an adult)

a =
9.

=
=

(a) A

(b) Number of men in the cinema = 240


= 150
P(the person is a man) =
=
(c) P(the person is a girl) =

(b) A B

(d) Number of female in the cinema = 240 + 80


= 170
P(the person is a female) =
=

3/7

11. Given that ABC is similar to ADB,


=
=
AC =
CD =
=
=

SECTION B
Ratios of corresponding
sides are equal

4 cm
AC AD
41
3cm

=
=

(b)

Sub x = 0 into line AB,


y = 0 +5
= 5
A (0, 5)
(b) When the line AB cuts the x-axis, y-coordinate = 0,
Sub y = 0 into line AB,

+5 = 0

(c) Gradient of CD = gradient of AB =


y intercept = 3
Equation of CD y =

(a + b)2 = a2 + 2ab + b2
256 = a2 + 2ab + b2
2
a + b2 = 256 2ab
6ab = 54
2ab = 18
Sub (2) into (1),
a2 + b2 = 256 18
= 238
4(a2 + b2) = 4(238)
= 952

(1)
(2)

(c) (i) % increase in his basic salary from 1999 to 2000

= 5
x = 15
B = (15, 0)
Equation of the line passing through B parallel to y-axis
x = 15

Area of ABCD =
=
=
=

12. (a) When the line AB cuts at y-axis, x-coordinate = 0,

(d)

13. (a)

+3

When the lines are


parallel, their gradients
are the same.

100

(ii) His bonus = $38400 $34000


= $4400
2% $4400
1% $2200
100% $220000
Total sales he made in year 2000 = $220000

Base Height
AD OB
2 15
30 unit2

4/7

14. (a) Revenue = 120 8


= $960

(ii) Middle number =

= 13th position

(b) Number of tickets sold after the increase = 120 5 5


= 110
Revenue when the price of each ticket is $12 = 12 (110)
= $1320

Median number of cars = 2


(iii) Modal number of cars = 1
16. (a) Circumference of circle = 2 r

(c) Revenue = Price of ticket Number of patrons


= (8 + 2x) (120 5x)
= 960 40x + 240x 10x2
= 960 + 200x 10x2 (shown)

= 2 3.14 3.4
= 21.352 cm

(d) When the revenue is $1950,


960 + 200x 10x2 = 1950
96 + 20x x2 = 195
x2 20x + 99 = 0
(x 11)(x 9) = 0
x = 11 or 9

Arc length of AB =

1
7

2
6

3
5

4
2

5
0

6
1

=
= 1.92

.
.

AOB = 0.2201 360


= 79.243
79.2 (1 d.p.)

(b) Area of sector AOB =

(b) (i) Mean number of cars


=

circumference of circle

= 0.2201

0
4

15. (a)
Number of cars
Number of families

th

Finding the 13th


value from the
frequency table

area of circle

r2
.

=
3.14 3.42
= 7.99
8 cm2 (nearest cm2)

5/7

17. (a) Radius of Cone = 45


= 7.5 cm
Height of cone = 45 2
= 22.5 cm
Volume of cone = r2h
= 3.14 7.52 22.5
= 1324.6875 cm3
Volume of hemisphere =
= 3.14 22.53
= 23844.375 cm3
Volume of remaining solid = Volume of hemisphere Volume of cone
= 23844.375 1324.6875
= 22519.7 cm3
(b) Surface area of cone = rl
= 3.14 7.5 23.717
= 558.537 cm2
Surface area of hemisphere = 2 r2
= 2 3.14 22.52
= 3179.25 cm2
Total surface area of the remaining solid
= Surface area of cone + surface area of hemisphere
+ base area of hemisphere base area of cone

18. (a) P(the first pen is blue) =


(b) P(the 1st and 2nd pen are blue) =

=
(c) P(the next pen drawn will be black) =

19. (a)

= x2

= x2
2x + 3x 4 = 6x 12
5x 4 = 6x 12
x = 8
(b) Number of days =
= 59 days
1 man
59 men
men
9.8 men

8h per day
8h per day
8h per day
8h per day

59 days
1 day
6 days
6 days

10 men are needed.

= 558.537 + 3179.25 + 3.14 22.52 3.14 7.52


= 5150.787
5150 cm2 (3 sig. fig.)

6/7

20. (a) O = {7, 9, 11, 13, 15, 17, 19, 21, 23, 25, 27, 29}

21. (a) a = 0, b = 4

P = {7, 11, 13, 17, 19, 23, 29}

x
y

M = {10, 15, 20, 25, 30}

(b)

O
9, 21,
27

7, 11,
13,17,
19,23,
29

15,25
6,8,12,14
16,18,22,24
,26,28

10,20,
30

(b) From the diagram, n(O M) = 2, n(O M) = 15

(c) Minimum value of y = 6.25


(d)

2x2 + 2x 14 = x2 + x 7
x = 2.8, 1.8

7/7

SECTION A (50 MARKS)


1.

Using your calculator, find the value of


(a) 4 2 , leaving your answers in 2 decimal places
3
3
(b) 1 2 11 19 , leaving your answers in fraction
5
20 15

MATHEMATICS
(Secondary Two)

2.

[1]
[1]

In the diagram below, ABC is similar to PQR. Find the length of BC and AC.
[4]
P

End-Of-Year Examination 2014

(2 hours)

3.6cm

4.8cm

3cm

VIII
Answer all questions in Section A AND Section B.
If the degree of accuracy is not specified in the question, and if the
answer is not exact, give the answer to three significant figures.
Give answers in degrees to one decimal place.
The number of marks is given in brackets [ ] at the end of each
question or part question.
The total number of marks for this paper is 100.
The use of an electronic calculator is expected, where appropriate.
Omission of essential working will result in loss of marks.
You are reminded of the need for clear presentation in your
answers.

6 printed pages, including this cover page and answers sheet

3.

4.2cm

A model of a plane is made to a scale of 2 cm to 5 m.


(a) If one wingspan of a plane is 15 m, find the wingspan of the model plane.
[1]
(b) Given that the area scale of the wingspan is 1: n. What is the value of n?
[2]

4.

Solve the simultaneous equations.


5x 8y = 7
2y x = 1

[4]

1/6

5.

6.

(a) Factorise 16 + 4x 6x2 completely.

[2]

(b) Using the results above, evaluate 16 4( 8 2 ) 6 (8 2 ) 2 .


3
3

[2]

Solve the following quadratic equation


(a) (x + 3)(x 4) = 8
(b) (2s + 3)2 = 25

7.

10. The diagram shows the graph of y = 2x2 4x 6. The graph cuts the x-axis at A
and B and the y-axis at C. Write down

(a) Given that Z = bn 1 , express n in terms of Z, m and b.


mn
(b) Hence, find the value of n given m = 5, Z = 3 and b = 2.

[2]
[2]

[3]
[1]

8.

The equation of line AB is 3y = 2x 5 and equation of line BC is 3x y = 4.


Find the coordinates of B.
[4]

9.

The equation of a line, L1 is 3y + 2x = 9. Find

(a) the coordinates of A, B and C


(b) the line of symmetry of the graph

[3]
[2]

11. Given the diagonal of a rectangle is 5 cm and that the length of the rectangle is
4 cm, calculate the area of the rectangle.
[3]

12. The diagram shows a sketch of the line y = ax + 3. Given that the point A(5, 8)
lies on the line and the point B is the where the line intersect the x-axis, find

(a) The coordinates of the point when L1 cuts the x-axis.

[1]

(b) The value of h given that the point (8, h) lies on L1.

[1]

(c) The point of intersection when L1 intersects with the line y = 13.

[1]

(a) the value of a


(b) the coordinates of B

[2]
[2]

2/6

13. The table below shows the number of errors made by 20 students when reciting a
passage.
Number of errors
Number of students

0
5

1
3

2
x

3
4

4
y

14. (a) 2 groups of students set off to a neighbouring country to build a toilet as part
of CIP at 2 villages. The first group consist of 24 students, working 8 hours a
day for 15 days.

5
2

(a) If the median is given to be 2.5, find the value of x and y.

SECTION B (50 MARKS)

[3]

The second group consist of 16 students, working 9 hours a day, how many
days will the second batch be doing their CIP?
[2]

(b) Using the values of x and y in (a), find


(i) mode
(ii) mean

[1]
[2]

(b) The Central Provident Fund (CPF) is a social security savings plan for
Singaporeans and the permanent residents. The table below gives the
contribution rate in the year 2000 for the various age groups:
Age Group
Y (years)

Employers
Contribution
(% of
salary)

Employees
Contribution
(% of salary)

Total
Contribution
(% of salary)

y < 35

10
10

20
20

30
30

45 y < 55

10

20

12.5

7.5

35 y < 45
55 y < 60
60 y < 65
Y

65

Credited to
Ordinary
Account
(% of
salary)
24
23

Credited to
Medisave
Account
(% of
salary)
6
7

30

22

16.5

8.5

9.5

1.5

(i) John is 28 years old and he works as a supervisor in a factory earning a


monthly income of $1650. Calculate his take home pay after deducting CPF.
[1]
How much will be credited into his Ordinary Account per month?

[1]

(ii) Mr Nathan is 56 years old and he works as a Senior Manager earning $8500
per month. Calculate the amount credited into his Ordinary Account for the
whole of year 2000 assuming he also gets an additional 2 months year-end
bonus.
[2]

3/6

15. A polygon ABCDEF has an interior angle of 70 whereas the rest are x each.
(a) How many sides does the polygon has?

[1]

(b) Find the value of x.

[3]

19. Andy and Rachel started together at the same constant speed x km/h on a 7 km
walk. After 1 km, Rachel increased her speed by 1 km/h, travelling at this speed
for the remaining 6 km. Andy walked at the constant speed x km/h till the end.
(a) Write down, in terms of x, the time taken by Andy and Rachel respectively
to complete the 7 km walk.
[2]

16. (a) Show that


(u 3)
u4
5u 2
18u 2 12u
3
5 .

2
2
3
6u
5(u 9) 15(u 3) 3u 7u 6
u4 u3
[3]

u4
5u 2
18u 2 12u

0.
2
6u 3
5(u 9) 15(u 3) 3u 7u 6

(b) Hence, solve (u 3)


2

[2]

17. (a) Simplify the expression


( p 3q ) 2 (3q p ) 2 .
(b) Factorise 2my 4mx 3ny 6nx.

[2]
[2]

18. Given that = {a, b, c, d, e, f, g}, A = {a, b, e, f} and B = {a, e},

(b) Given that Rachel finished the walk half an hour earlier than Andy, form an
[2]
equation in x and show that it reduces to x2 + x 12 = 0.
(c) Calculate the time, in hours, Rachel takes to complete her walk using the
chosen value. Explain also why you need to reject one of the answers. [2]
20. (a) The table below shows the age in months of the books that a student has
collected.
Age in months (m)
0 < m 10
10 < m 20
20 < m 30
30 < m 40
40 < m 50
50 < m 60

Number of books (f)


10
15
25
20
30
15

(i) State the modal class.

[1]
[3]

(a) Write down, in set notation, the relationship between set A and B.

[1]

(ii) Calculate an estimate mean age of the books.

(b) List the element of B A

[1]

(iii) If the student randomly picks a book from his collection, what is the
probability that the age of the book is older than 30 months?
[1]

(c) Draw a Venn diagram to represent the sets.

[2]
(b) The distribution of marks scored by pupils of a class is as follow:
Marks
No. of students

4
7

5
5

6
6

7
3

8
x

9
8

10
7

If the median mark is 7, write down all the possible values of x.

[2]

4/6

21. Answer the whole of this question on the sheet of graph paper provided.
PQRS is a square of side 9 cm and PTUV is a rectangle.
Given that PT = UV = VS = x cm,
P
T
Q

22. An open container is made up of a hollow cylinder and a hollow conical base of
radius 6 cm. The height of the cylinder is 2.5 cm and the height of the conical
base is 7 cm. Slant height of the cone is 9.2 cm. Taking = 3.142 and correcting
your answers to 2 decimal places where applicable, calculate
(a) the height of the water level from the tip of the cone when 300 cm3 of water
is poured into the container.
[2]
(b) the area where the water is in contact with the container.

[2]

3
(c) the radius of the hemispherical container if the volume is the size of the
8
V

container.

[3]
3

If water is leaking at a rate of 0.2 cm per second from the tip of the cone into a
hemispherical container below,
S

(d) find the time taken, in minutes to completely fill up the hemispherical
container.
[1]

(a) show that area of the rectangle PTUV is ( 9x x2) cm2.

[1]
6

(b) Given that y = 9x x2, copy and complete the table below:
x
y

0
0

1
8

3
18

7
14

8
8

2.5

9
0

[1]
(c) Using a scale of 2 cm to represent 1 unit on the x-axis and 1 cm to represent
1 unit on the y-axis, draw the graph of y = 9x x2, for values of x in the
range 0 x 9.
[2]
(d) From your graph, find the maximum area of the rectangle PTUV.

9.2

[1]

(e) Using your graph, estimate the values of x for which the area of PTUV is
[1]
10 cm2.

5/6

ANSWERS (dont peek till youve tried the question!)


(a) 1.80

2.

BC = 3.55cm, AC = 4 cm

3.

(a) 6 cm

4.

x = 3, y = 1

5.

(a) 2(4 + 3x)(2 x)

(b) {b, f}

(c)

(b) 2 53
60

1.

18. (a) B A

a,
e

b, f

(b) 62500

19. (a) Andys time =

(b) 400
6.

(a) x = 4 or 5

7.

(a) n = zm 1
bz

8.

B(1, 1)

9.

(a) ( 4 1 ,0)
2

(b) s = 4 or 1
1
(b) n = 3
5

(b)

1
6
7
hrs, Rachels time =
hrs
x x 1
x

(c) 1 5 hours
6
20. (ai) 40 < m 50 (aii) 32.8

(aiii) 13
23

(b) x = 1, 2, 3, 4, 5

7
3

21. (b) 14, 20, 20, 18


(d) 20.3

(c) (15, 13)


10. (a) A = (1, 0), B = (3, 0), C = (0, 6) (b) x = 1

22. (a) 7.32 cm


(c) 4.61 cm

11. 12 cm2
12. (a) a = 1

c, d, g

(e) 1.3 or 7.7


(b) 185.46 cm2
(d) 17.08 min

(b) (3, 0)

13. (a) x = 2, y = 4 (bi) mode = 0

(bii) mean = 2.5

14. (a) 20 days

(bi) $1320; $396

(bii) $10 115

15. (a) 6 sides

(b) 130

16. (b) u = 5.5


17. (a) 12pq

(b) (y + 2x)(2m 3n)

6/6

SECTION A
1.

(a)

= 1.804
1.80 (2 d.p.)

(b) 1

= 1
=

2.

MATHEMATICS
(Secondary Two)

Given that ABC is similar to PQR


=
=
.
BC2 = 12.6
BC = 3.549
3.55 cm (3 sig. fig.)

End-Of-Year Examination 2014


VIII

=
.

Worked Solutions

Ratios of corresponding
sides are equal

.
.

3
AC =
.
= 4 cm

(Dont peek till youve tried the question!)


3.

(a)

(b)

7 printed pages

Ratios of corresponding
sides are equal

2 cm : 5 m
2 3 : 15 m
= 6 cm
Wingspan of the model plane is 6cm.
2 cm
1 cm
1 cm2
1 cm2

:
:
:
:

5m
250 cm
(250 cm)2
62500 cm2

Area scale 1 : 62500

1/7

4.

5x 8y = 7
2y x = 1
x = 2y 1
Sub (2) into (1),
5(2y 1) 8y
10y 5 8y
2y
y
When y = 1,
x = 2 1
= 3
x = 3 and y = 1

5.

=
=
=
=

(1)
(2)

7
7
2
1

4 8

=
=
=
=
=

bn 1
bn 1
bn + zn
zm + 1

n =
n =

6 8

(x + 3)(x 4)
x 4x + 3x 12
x2 x 20
(x 5)(x + 4)
x = 5
or
2

(b)

z
z(m n)
zm zn
zm + 1
n(b + z)

(b) When m = 5, z = 3 and b = 2,

n =

=2 2

n =
8

=2 6
= 400
(a)

(a)

n =

(a) 16 + 4x 6x2 = 2(3x2 2x 8)


= 2(x 2)(4 + 3x)
= 2(2 x)(4 + 3x)
(b) 16

6.

7.

(2s + 3)
4s2 + 12s + 9
4s2 + 12s 16
s2 + 3s 4
(s 1)(s + 4)
s1 = 0
or
s = 1

=
=
=
=

8
8
0
0
x = 4

= 25
= 25
= 0
= 0
= 0
s+4 = 0
s = 4

4
30

3 8

8.

3y = 2x 5
3x y = 4
y = 3x 4

(1)
(2)

Sub (2) into (1),


3(3x 4)
9x 12
7x
x

=
=
=
=

2x 5
2x 5
7
1

When x = 1,
y = 34
= 1
B = (1, 1)

2/7

9.

(a) Sub y = 0 into 3y + 2x = 9,


2x = 9
x =

When the line cuts the xaxis, y-coordinate = 0.

(b) Mid-point of AB =
=1
Line of symmetry: x = 1

= 4
Coordinates:

(b) Given that (8, h) lies on L1,


sub x = 8 and y = h into 3y + 2x = 9,
3h + 2(8) = 9
3h = 9 16
h =
(c) When L1 intersects with line y = 13,
sub y = 13 into 3y + 2x = 9,
3(13) + 2x =
2x =
=
x =

9
9 39
30
15

Coordinates are (15, 13)

11. Breadth =
=
=
=
Area =
=
=

4
5
25 16
9
3 cm
Breadth Length
34
12 cm2

Using Pythagoras theorem

12. (a) Since the A(5, 8) lies on the line,


sub x = 5, y = 8,
8 = 5a + 3
5 = 5a
a = 1
(b) When y = 0, a = 1,
0 = x + 3
3 = x
Coordinates of B: (3, 0)

10. (a) When the curve cuts the y-axis, x = 0,


y = 2(0)2 4(0) 6
y = 6
C: (0, 6)
When the curve cuts the x-axis, y = 0,
2x2 4x 6 = 0
(x 3)(x + 1) = 0
x = 3
or
x = 1
Coordinates of B: (3, 0), Coordinates of A: (1, 0)

3/7

13. (a)

Number of errors = 20
5 + 3 + x + 4 + y + 2 = 20
x+y = 6
(1)
Given that the median is 2.5 mean of 10th and 11th = 2.5
10th value 2, 11th value 3
x=2

=
= 2.25

24 students
1 student
1 student
1 student

8hrs per day


8hrs per day
1hr per day
9hrs per day

15 days
(15 24) days
(15 24 8) days


days

(b) (i) Mode = 0


14. (a)

16 students 9hrs per day


= 20 days

Sub x = 2 into (1), y = 4.


x = 2 and y = 4

(ii) Mean =

SECTION B

Fewer students
require more
days, thus you
multiply.

days

(b) (i) CPF contribution for 28 years old 20%


His take home pay = (100 20)% 1650
= 80% 1650
=
1650
= $1320

More students
require less
days, thus you
divide.

Amount credited to his Ordinary account =


1650
= $396
(ii) Nathans total salary for year 2000 = $8500 14
= $119000
.
Amount credited to his Ordinary account =
119000
= $10115
15. (a) Number of sides = 6
(b) Total sum of interior angles in 6 sided polygon = (6 2) 180
720 = 70 + 5x
Sum of interior angle
5x = 650
in n-sided polygon
x = 130
= (n 2) 180

4/7

(b)

(b) B = {b, c, d, f, g}
B A = ,

18. (a) B A

16. (a)

(c)

(shown)

= 0

a,
e

b, f

Using the part (a)


answer to solve

c, d, g

= 0
=

5(u + 4)
5u + 20
2u
u
17. (a) (p 3q)2 (3q p)2
= p2 6pq + 9q2 (9q2 + 6pq + p2)
= 12pq

=
=
=
=

19. (a) Andys time taken =

3(u + 3)
3u + 9
11
5.5

hour

Rachels time taken =


(b)
2

Using (a b) = a 2ab + b

hour

=
=

(14 x)(x + 1)
14x + 14 x2 x
x2 x + 12
x2 + x 12

(b) 2my + 4mx 3ny 6nx


= 2m(y + 2x) 3n(y + 2x)
= (y + 2x) (2m 3n)
(c)

=
=
=
=

(7x + 1)(2)
14x + 2
0
0 (shown)

x2 + x 12 = 0
(x 3)(x + 4) = 0
x3 = 0
or
x = 3

x+4 = 0
x = 4
(rejected speed cannot be negative)

5/7

Modal class = the class


that occurs most often

20. (a) (i) Modal class = 40 m 50

(b) Median

=7

4, , 4, 5, ., 5, 6, .6, 7, 7, 7, 8, .,8, 9, .,9, 10, , 10

(ii)
Age in months (m)
0 < m 10
10 < m 20
20 < m 30
30 < m 40
40 < m 50
50 < m 60
Estimate mean age =
=

Number of books (f)


10
15
25
20
30
15

Mid-value (x)
5
15
25
35
45
55

32.826
= 32.8 (3 sig. fig.)
(iii) Number of books older than 30 months = 20 + 30 + 15
= 65
P(the age of the book that is older than 30 months) =
=

6
Case (1)

Case (2)
Case (1), 1st 7 is the median
7+5+6 = 2+x+8+7
18 = 17 + x
x = 1
Least possible of x = 1

Number of elements on the


left hand side of the median
= Number of elements on
the right hand side of the
median

Case (2), 3rd 7 is the median


7+5+6+2 = x+8+7
20 = 15 + x
x = 5
Greatest possible of x = 5
Possible values = 1, 2, 3, 4, 5
21. (a) Area of PTUV = PT PV
= x(9 x)
= 9x x2
(b)
x
0
1
2
3
y
0
8
18
14

4
20

5
20

6
18

7
14

8
8

9
0

6/7

(c)

=
= 0.3189 cm
0.32 cm (2 d.p.)
Height of the water level from the tip of the cone = 7 + 0.32
= 7.32 cm
(b) Area where the water is in contact with the container
=
2
= 6 9.2
2 6 0.3189
= 185.462 cm2
185.46 cm2 (2 d.p.)
(c) Volume of hemisphere =

36 7

84

36 2.5

90

=
r3 =
r = 4.608
4.61 cm (2 d.p.)
(d)

Time taken =
=

Volume of water poured in = 300 cm3


Volume of cone =
6 7
= 84
Volume remaining in cylinder = 300 84
Height in cylinder =

.
.

= 1047.76 s
= 17.079 minutes
17.1 minutes (3 sig. fig.)

(d) Maximum area of PTUV = 20.3 cm2


(e) When y = 10, x = 1.3 or 7.7
22. (a)

7/7

SECTION A (50 MARKS)


1.

Evaluate 8 1 2 7 9 2
15
5
6

2.

Find the value of the following:


(a)

MATHEMATICS
(Secondary Two)

(b)

End-Of-Year Examination 2014


(2 hours)

1
1
3
3

3
125 4
1 2
4
2 3
1 2
4
2 3

[2]

[1]
[2]

3.

At a car wash, 8 men working 6 hours can wash all the cars that come to the
car wash. How many men are required to wash the cars in 4 hours?
[2]

4.

Solve the simultaneous equations:

IX
Answer all questions in Section A AND Section B.
If the degree of accuracy is not specified in the question, and if the
answer is not exact, give the answer to three significant figures.
Give answers in degrees to one decimal place.
The number of marks is given in brackets [ ] at the end of each
question or part question.
The total number of marks for this paper is 100.
The use of an electronic calculator is expected, where appropriate.
Omission of essential working will result in loss of marks.
You are reminded of the need for clear presentation in your
answers.

3x + 8y = 13
1
x 2y 4
2
5.

[4]

Simplify the following expressions.


(a) 2(t 2 1) (t 1)(t 2)
2

[2]

(b) 66ab c
3bc 3

[3]

6 printed pages, including this cover page and answers sheet

1/6

6.

Factorize completely the following:

9.

(a) 4a2 9
(b) 9x2 12x + 4

[2]
[2]

AB, BC and CD are adjacent sides of a regular polygon. Given that


CAB = 18, calculate

B
A

7.

Find the values of x, y and z in the figure below.

18

[3]

C
20

120

y 4y

8.

(a)
(b)
(c)
(d)

Solve the following equations:

3x 5 5 x 4

2
6
5
1
2
(b)
x 5 2 x 3
4
5

(a)

an interior angle of the polygon


an exterior angle of the polygon
the number of sides of the polygon
ACD

[1]
[1]
[1]
[1]

[2]
10. Study the following sequence: 2, 1, 6, 13, ..
[2]

(a) Write down the next number in the sequence.


(b) Write down the 8th term in the sequence.
(c) The nth term in the sequence is 141. What is the value of n?

[1]
[1]
[1]

2/6

11. Triangles ABC and DEF are similar. Calculate the value of x. All given
dimensions are in metres.

14. In that diagram, AB = 2x cm, AC = 3cm, CD = x cm and BD = h cm.


[2]
B

D
2x cm

h cm

4.8

3.8

9
A

12. If

3cm

x cm

x
y 5 , express y in terms of x.

2
3 y

(a) Given that area of ABC = 9cm2, write down a pair of simultaneous
equations involving h and x.

[2]

(b) Solve the equations to find the values of h and x.

[3]

[3]

13. The shoe sizes of 19 boys in a group are as follows:


7
6
(a)
(b)
(c)
(d)

6
9

10
7

9
6

6
7

8
8

10
8

10
11

5
7

State the modal shoe size.


Find the mean shoe size.
Find the median shoe size.
If another boy joins the group and his shoe size is 8, what is the new
median shoe size?

[1]
[1]
[2]
[2]

3/6

SECTION B (50 MARKS)


15. (a) Find the value of

17. If 1 is added to both the numerator and the denominator of a fraction, the fraction
becomes 3 . If 2 is subtracted both from the numerator and the denominator of
4

818

[1]

6 5000
5

(b) A street directory is drawn to a scale of 1 : 80 000.


(i) An expressway is represented by a line of length 15 cm.
Calculate the time taken in minutes by a taxi moving at 72 km/h to
travel from one end of the expressway to the other.
[3]

2
3

the fraction, it becomes . Find the fraction.

[5]

18. The diagram below shows a model with a hollow cone inside a solid hemisphere.
The radius of the solid hemisphere is 12 cm. Given that the cone has a circular
base with diameter of 10 cm a slant length of x cm as indicated in the diagram.

10 cm

(ii) The actual area of a golf course is represented by 1.76 km2.


Calculate the area on the street directory which represents the golf
course, giving your answer in square centimetres
[2]

x cm

12 cm

16. Mr. Lee bought a new car priced at $105 000 at a motorcar exhibition. He pays
20% of the price as a down payment and borrowed the remaining from the bank.
(a) Calculate the remaining amount that he borrowed from the bank.

[1]

(b) The bank charges simple interest at a rate of 2.5% per annum.

(a) Show that the value of x is 13.

[3]

(b) Find the volume of this model, correct to 2 decimal places.

[3]

If Mr. Lee takes a 7-year loan from the bank, calculate


(i) the total interest he has to pay,
(ii) his monthly installment over the next 7 years.

[1]
[2]

(Take to be 3.142.)

Instead of buying a new car, Mr. Lee decides to save $20 000 in the bank.
The saving plan offers a compound interest of 2.23 % per annum.
Find the total amount of money that Mr. Lee receives after 2 years.

[2]

4/6

19. (a) Tom ran the 20 km of a marathon race at an average speed of x km/h. Write
down, in terms of x, an expression for the number of hours it took him to
complete the race.
[1]
(b) Charles ran the race at an average speed which was 5 km/h greater than
Toms speed. Write down, in terms of x, an expression for the number of
hours which Charles took.
[1]
1
h
, write down an
(c) Given that the difference between the two times was
3
[3]
equation in x and show that it reduces to x2 + 5x 300 = 0.

21. (a) It is given = x : x is a whole number and 1 x 13.


A = x : x is a prime number
B = x : x is a factor of 30
(i) List the elements of A B and A B.
(ii) Find n(A)
(b) Shade the regions in the Venn Diagram that represents (X Y).

[2]
[1]
[2]

(d) Solve the equation x2 + 5x 300 = 0 and find in hours and minutes, the time
it took Tom to complete the race.
[3]

20. (a) In a Mathematics test, the scores of the students are recorded as shown.

22. Answer the whole of this question on a sheet of graph paper.


25
30

37
14

14
27

47
29

11
32

27
38

40
45

23
21

9
34

29
28

(a) The following table of values is for the equation y = x2 3x + 3.

Construct a stem-and-leaf diagram to represent the scores of the students.


[2]
(b) Some students were asked how many hours they spend using the computer
after school. The results are as shown.

No of hours
No of students

0
6

1
10

2
3

3
4

4
3

x
y

2 1 0
m 7 3

1
1

2
1

3
3

4
7

Calculate the value of m.

[1]

(b) Using 2 cm to represent 1 unit on the x-axis and 1 cm to represent 1 unit on


The y-axis, draw the graph of y = x2 3x + 3 for x-axis ranging from 2 to 4.
[3]
(c) Write down the equation of the line of symmetry.
[2]

5
x

(i) Write down the largest possible value of x given that the mode is 1. [1]
(ii) Find the value of x given that the median is 2.5.

[3]

(iii) Calculate the value of x given that the mean is 2.

[2]

(d) Use your graph to


(i) find the value of y when x = 1.5,
(ii) find the coordinates when y is at the minimum.

[2]

5/6

ANSWERS (dont peek till youve tried the question!)


1.
2.

14. (a)

(b) h = 6, x = 5

57
94

(a) 13
20

(b) 1

5
18

15. (a) 0.178

(bi) 10 min

(bii) 2.75 cm2

16. (a) $84 000

(bi) $14700

(bii)$1175, $20 901.95

3.

x = 12

17. 8
11

4.

x = 5, y = 0.25

18. (b) 3305.38 cm2

5.

(a) t2 3t 4

19. (a) 20 h
x

(b) 22ac
9b
(a) (2a + 3)(2a 3)

7.

x = 60, y = 20, z = 40

8.

(a) 4

9.

(a) 144

(b) 36

(c) 10

(d) 126

1
15

10. (a) 22

(b)

20
h
x5

(d) 1h 20 min

20. (a)

6.

(b) (3x 2)

Stem
0
1
2
3
4

(b) 1
11

(bi) x = 9

Leaf
9
1 4 4
1 3 5 7 7 8 9 9
0 2 4 7 8
0 5 7
(bii)x = 12

(biii)x = 4

21. (ai) A B = {2, 3, 5}, A B = {1, 2, 3, 5, 6, 7, 10, 11, 13}

(b) 61

(aii) n(A) = 7

(c) n = 12

(b)

11. 7.2

2
12. y 3x 20
4 x2

13. (a) 7

= 9, h2 + (3 + x)2 = (2x)2

(b) 7 14
19

(c) 7

(d) 7.5
22. (a) m = 13

(c) x = 1.5

(di) y = 9.6

(dii) (1.5, 0.7)

6/6

SECTION A
1.

=5

=
2.

(a)

MATHEMATICS
(Secondary Two)

End-Of-Year Examination 2014

(b)

(Dont peek till youve tried the question!)

=
=

3.

IX
Worked Solutions

8 men
(86) men
men
12men

6 hours
1 hour
4 hours
4 hours

12 men are required to wash the car in 4 hours.


4.

3x + 8y = 13
x + 2y = 4
x =
Sub (2) into (1),
3 4
2 + 8y=
13.5 6y + 8y =
2y =
2y =
y =
x =

6 printed pages

More men require


fewer hours, thus
you multiply.

Fewer men require


more hours, thus
you divide.

(1)
2y

(2)

13
13
13 13.5
0.5
0.25
0.5 = 5
4

x = 5, y = 0.25

1/6

5.

(a) 2(t2 1) (t + 1)(t + 2) = 2t2 2 (t2 + 3t + 2)


= 2t2 2 t2 3t 2
= t2 3t 4
(b)

8.

(a)

= 2
= 2

= 2
15x 1 = 60
15x = 61
x =

(3)3 = 27

=
6.

Using a2 b2 = (a + b)(a b)

(a) 4a2 9
= (2a)2 32
= (2a 3)(2a + 3)

x =
(b)

Using (a b)2 = a2 2ab + b2

(b) 9x2 12x + 4


= (3x)2 2(3x)(2) + (2)2
= (3x 2)2

=
x =

7.

20

60

120

9.

(a) Interior angle of polygon, ABC = 180 18 18 (sum of angles in )


= 144
(b) Exterior angle = 180 ABC
= 180 144
= 36

y 4y

(c) Exterior angle in n-sided polygon =

x
x + 120 = 180
x = 180 120
= 60
5y + x + 20 = 180
5y = 180 20 60
5y = 100
y = 20
z = 180 60 4(20)
= 40

Sum of interior angle and


exterior angle = 180

36 =
36n = 360
n = 12

(sum of interior angles)

(sum of interior angles)

(sum of angles in )

Number of sides of the polygon = 12.


(d) ACD = 144 BCA
= 144 18
= 126

2/6

10. (a) 2, 2 + 3 = 1, 1 + 5 = 6, 6 + 7 = 13, 13 + 9 = 22


(b) 2, 2 + 3 = 1, 1 + 5 = 6, 6 + 7 = 13, 13 + 9 = 22, 22 + 11 = 33,
33 + 13 = 46, 46 + 15 = 61

(c) 5, 6, 6, 6, 6, 7, 7, 7, 7, 7, 8, 8, 8, 9, 9, 10, 10, 10, 11


Middle position =
= 10th position

th

8 term in the sequence is 61.


(c) General formula for nth term = n2 3
n2 3 = 141
n2 = 144
n = 12
11. Given ABC and DEF are similar.
=
Ratios of corresponding sides are
.
equal
=
6x = 9 4.8
x = 7.2 m
12.

(d) New median = mean of 10th value and 11th value


=
= 7.5
14. (a) Given area of triangle ABC = 9 cm2
= 9
(1)
h2 + (3 + x)2 = (2x)2 (2)

Using Pythagoras
theorem

= 9

=
3h =
h =
Sub h = 6 into (2),
2x2 =
4x2 =
2
3x 6x 45 =
(3x + 9)(x 5) =
x = 5
or

=
=
3x2 + yx2 = 4y 20
3x2 + 20 = 4y yx2
3x2 + 20 = y(4 x2)

13. (a) Modal shoe size = 7


(b) Mean shoe size =

Median = 7

(b) From (1),

y =

10th value

9
18
6
62 + (3 + x)2
36 + 9 + 6x + x2
0
0
3x = 9
x = 3 (rejected)

=
=

3/6

SECTION B

15. (a)
(b) (i)

(ii) Monthly installments =

1 : 80 000
1 cm : 80 000 cm
1 cm : 0.8 km
15 cm : 0.8 15 = 12 km
Length of expressway = 12 km
Time taken =

:
:
:
:
:

= 20000 1

= 20000 1.02232
=$20901.95 (2 d.p.)
17. Let x be the numerator and y be the denominator.

3y + 3 = 4x + 4
3y = 4x + 1
y =

0.8 km
(0.8 km)2
0.64 km2
1.76 km2
1.76 km2

The total amount of


investment for a principal of
$P for n years at a
compound interest of R%
per annum is $P 1

= hour
= 10 minutes
1 cm
(1 cm)2
1 cm2
.
cm2
.
2.75 cm2

=

= $1175
Total amount of money received after 2 years

0.178 (3 sig. fig.)

(ii)

(1)

=
If the simple interest on
$P for T years at R%
per annum is $I, then
.
I=

Area of map = 2.75 cm2


16. (a) Amount he borrowed from bank =
105000
= $84000
.

(b) (i) Total interest he has to pay = 7


84000
= $14700

2y 4 = 3x 6
2y = 3x 2

(2)

Sub (1) into (2),


2
= 3x 2
2(4x + 1) = 3(3x 2)
8x + 2 = 9x 6
x = 8
Sub x = 8 into (1),
y =
= 11
The fraction is

4/6

18. (a)

52 + 122 = x2
x = 5
12
= 13 (shown)

Using Pythagoras theorem

(b) Volume of hemisphere =


= (3.142)(12)3
= 3619.584 cm3
Volume of cone =
= (3.142)(5)2(12)
= 314.2 cm3
Volume of model = 3619.584 314.2
= 3305.384
3305.38 cm2 (2 d.p.)
19. (a) Time taken for Tom =
=

hours

(b) Charles speed = (x + 5) km/h


Time taken for Charles =

=
(c)

hours

(d) (x 15)(x + 20) = 0


x = 15
or
x = 20 (rejected speed cannot be negative)
Time taken for Tom to complete the race = hours
= 1 hour 20 minutes
20. (a)
Stem
Leaf
0
9
1
1 4 4
2
1 3 5 7 7 8 9 9
3
0 2 4 7 8
4
0 5 7
(b) (i) Largest possible value of x = 9
(ii) Since median is 2.5,
median = mean of 19th and 20th value
18 = 3 + 3 + x
18 = 6 + x
x = 12
(iii)

Mean =

= 2
40 + 5x = 52 + 2x
3x = 12
x = 4

Since mode = 1,
x <10.

Number of elements on
the left hand side of the
median = Number of
elements on the right
hand side of the median

=
300 = x2 + 5x
x2 + 5x 300 = 0 (shown)

5/6

21. (a) A = {2, 3, 5, 7, 11, 13}, B = {1, 2, 3, 5, 6, 10}

1 is not a prime
number.

22. (a)

A B = {2, 3, 5}

(i)

A B = {1, 2, 3, 5, 6, 7, 10, 11, 13}


(ii) A = {1, 4, 6, 8, 9, 10, 12}
n(A) = 7

(b)
x
y

AB
The union of sets A and B is the
set of elements which are in A,
or in B or in both A and B.
AB
The intersection of sets A and B
is the sets of elements which are
common to both A and B

Complement of A
(denoted as A)
Elements that
are in but not in
A

y = (2)2 3(2) + 3
= 4+6+3
= 13
2
13

1 0
7 3

1
1

2
1

3
3

4
7

(b) (X Y)

(c)

y = 1.5

(d) (i) When x = 15, y = 9.8


(ii) (1.5, 0.4)

6/6

SECTION A (50 MARKS)


1.

The square of a number is 3 times as large as the number itself. Find the possible
numbers.
[3]

2.

A map is drawn to a scale of 1 : 50 000.


(a) An airport runway is represented by a line of length 4.53 cm on the map.
Calculate the actual length of the runway, giving your answer in kilometres,
correct to three significant figures. .
[2]

MATHEMATICS
(Secondary Two)

End-Of-Year Examination 2014


(2 hours)

(b) The actual area of the airport is 11.68 km2.


Calculate the area on the map which represents the airport, giving your
answer in square centimeters, correct to one decimal place.
[2]
3.

A polygon has n sides. Four of its exterior angles are 26, 31.5, 43 and 55.5 and
the remaining exterior angles are each equal to 8.5. Find the value of n.
[2]

4.

Factorise completely

X
Answer all questions in Section A AND Section B.
If the degree of accuracy is not specified in the question, and if the
answer is not exact, give the answer to three significant figures.
Give answers in degrees to one decimal place.
The number of marks is given in brackets [ ] at the end of each
question or part question.
The total number of marks for this paper is 100.
The use of an electronic calculator is expected, where appropriate.
Omission of essential working will result in loss of marks.
You are reminded of the need for clear presentation in your
answers.

5 printed pages, including this cover page and answers sheet

5.

(a) 20a3 4a2b 5a + b


(b) 9x2 12x 32

[3]
[1]

Simplify
3
2
4
3
(a) 4 x yz 15 x 5 x
3
2
( 2 xy )
7( xz )
21 yz 3

[3]

(b)
6.

7.

4
3

2x 3 6 4x

Given y varies inversely with x and when y = 2, x = 8. Find the equation


connecting x and y.

[2]

[2]

The diagonal of a rectangle is 17cm and its width is 8cm. Calculate the area
of the rectangle.
[3]

1/5

8.

9.

7 solid spheres are melted to form a pyramid which has a hexagonal base.
If the radius of each sphere is 3cm and the area of the hexagonal base of the
pyramid is 72cm2, calculate the height of the pyramid, giving your answer
correct to three significant figures.
[3]

13. In the diagram, O is the centre of a circle with radius 7.5cm. The points A, B and
C are on the circumference and AOB = 140.
C

Given that x + y = 7 and xy = 18,


(a) Show that x2 + y2 = 85 and hence or otherwise, evaluate
(b) (x y)2

[2]
[2]

140

MR 2 and hence find the value of


10. Make M the subject of the formula P = M
P
M when P = 4 and R = 2.

[4]

11. In the right-angled triangle PQR, S is a point on the side PQ. Given that
SQ = 24cm, PR = 30cm and SR = 34cm, calculate

(b) Find the area of the major sector AOBC, giving your answer to three
decimal places. [Take = 3.142]

30

(a) Calculate the length of the minor arc AB, giving your answer correct to two
significant figures.
[2]
[2]

14. The pie chart shows the number of visitors present in an exhibition.
6900
Malaysian

34

Japanese
125

24

(a) PS
(b) QR

75

Europeans

[1]
[2]

Others

(a) How many Europeans visited the exhibition?


(b) Find the value of x.
(c) If a visitor is selected, find the probability that the visitor is

12. The set of data given below shows the test scores of 20 students in a class.
12, 11, 12, 10, 9, 11, 11, 10, 12, 13, 10, 13, 12, 9, 11, 12, 10, 9, 13, 12
(a) Find the mean score.
(b) What is the median score?
(c) What is the modal score?

115

[1]
[1]
[1]

(i) Japanese
(ii) European
(iii) Malaysian

[1]
[2]

[3]

2/5

SECTION B (50 MARKS)

20. The following are the travel time, in minutes, from home to school of 28 students
represented in a stem and leaf diagram.

15. (a) Solve the equation 3(2x2 1) = 7x

[2]

2
2
(b) Simplify (3 2a) 2(9 4a )
9ab
6b 2 c

[3]

16. Solve simultaneous equations


3(x 2) + 1 ( y 1) = 0
2
1
1
x 2y 2
4
2

[4]

[2]

19. A group of people were asked to state the number of cars each person has. The
findings were tabulated below.
1
12

2
x

2
6
2
3
3
5

5
8
5
5
4

7 8 9
5 5 7 8 9
8 8
7

(a) What is the modal travel time?

[1]

(b) Find the median travel time of these students.

[2]

[1]

18. If y varies directly as x2 and the difference in the values of y when x = 4 and
when x = 2 is 36, find the value of y when x = 1.
[4]

0
7

2
5
2
0
0
0

(c) Calculate the mean travel time taken by students from home to school. [2]

17. Given that 2 x 7 and 8 y 2, find


(a) the least value of y x
1
(b) the greatest value of 2
x y2

No of cars
No of people

1
2
3
4
5
6

Row
1
2
3
4
5
6

P
42
342
3342
33342
333342
3333342

Q
16
1156
111556
11115556
p
q

Sum of digits of Q column


7
13
19
25
r
s

(a) Write down the values of p, q, r and s.

3
2

(a) If the mode is two, sate the smallest possible of x.


(b) Hence, find the mean and median.

21. Study the table below shows a sequence of squares.

[2]
2

[1]
[2]

(b) Find the sum of digits in the answer 333333334 .

[1]

(c) Find the sum of digits in the Q column in the nth row in terms of n.

[2]

3/5

22. Given that = {x : x is an integer and 3 x 20}


A = { x : x is a prime number}
B = { x : x is an odd number}
(a)
(b)
(c)
(d)
(e)

List the elements of A


List the elements of B
List the elements of A B
State the relationship between A and B.
List the elements of (A B)

24. In the figure below, a solid metal cone of radius 5cm and height 12.3cm is placed
on the bottom of a cylindrical container whose base radius is 10 cm. Water is
poured into the container until the top of the cone is just covered, and the cone is
then removed.
[1]
[1]
[1]
[1]
[1]

23. Answer the whole of this question on a sheet of graph paper.


The variables x and y are connected by the equation
1
y x 2 2x 1
2

Taking = 3.142, calculate


(a) the total surface area of the cone
[3]
(b) the volume of water in the cylindrical container
[2]
(c) the final depth of the water in the cylindrical container after the cone is
removed.
[2]

Corresponding values of x and y are given in the following table.


x
y

2 1 0
5 p 1

1
2.5

2
3

3
2.5

4
q

5
6
1.5 5

(a) Calculate the value of q and p.

[1]

(b) Using 2 cm to represent 1 unit on the both axis, draw the graph of
1
y x 2 2x 1
2
for values of x in the range of 2 x 6.
(c) Use your graph to
(i) find the value of y when x = 3.5
(i) find the values of x when y = 3

12.3

10

[2]
[1]
[2]

2
(d) On the same axes, draw the graph of y x 2 . Hence write down the
3
1 2
2
solutions of the simultaneous equations y x 2 x 1 and y x 2 .
2
3

[3]

4/5

ANSWERS (dont peek till youve tried the question!)

17. (a) 15

1.

18. y = 3x2, y = 3

0 or 3

(b) 1
4

2.

(a) 2.27 km

(b) 46.7 cm

3.

n = 28

4.

(a) (5a b)(2a + 1)(2a 1)

5.

(a) xz
18 y 3

6.

16
y
x

7.

120 cm2

8.

33.0 cm

9.

(a) -

(b) (3x + 4)(3x 8)

11
(b)
2(2 x 3)

19. (a) 13

(b) mean = 1.29, median = 1

20. (a) 35

(b) 36

21. (a) P = 1111155556, Q = 111111555556, R = 31, s = 37


(b) 55

(c) 6n + 1

22. (a) A = {3, 5, 7, 11, 13, 17, 19}


(c) {3, 5, 7, 11, 13, 17, 19}

(b) B = {3, 5, 7, 9, 11, 13, 15, 17, 19}


(d) A is a subset of B

(e) {3, 5, 7, 9, 11, 13, 15, 17, 19}


23. (a) p = 1.5, q = 1

11. (a) 16 cm

(b) 50 cm

12. (a) 11.1

(b) 11

13. (a) 18 cm

(b) 108.006 cm2

14. (a) 4500

(b) x = 45
(cii) 5
24

(ci) y = 1.85

(cii) 1.45, 5.45

(d) x = 0.4, y = 1.70 or x = 4.9, y = 1.30

(b) 121

24. (a) 287 cm3

2
10. M P
,M=2
2
( R P)

(ci) 25
72

(c) 36.4

(b) 3540 cm3

(c) 11.3 cm

(c) 12

(ciii) 23
72

15. (a) x = 1 1 or 1 (b) 3a (3 2a )


2
3
4bc (3 2a )
16. x = 2, y = 1

5/5

SECTION A
1.

Let the number be x


x2 = 3x
x2 3x = 0
x(x 3) = 0
x = 0
or
x3 = 0
x = 3
The possible numbers are 0 or 3.

MATHEMATICS
(Secondary Two)

2.

(a)

End-Of-Year Examination 2014


X

Map
1
1 cm
1 cm
4.53 cm

:
:
:
:
:

Actual
50 000
50 000 cm
0.5 km
4.53 0.5
= 2.265 km
2.27 km (3 sig. fig.)

Actual length of the runway = 2.27 km


(b)

Worked Solutions
(Dont peek till youve tried the question!)

1 cm
(1 cm)2
1 cm2
.
cm2
.

:
:
:
:

0.5 km
(0.5 km)2
0.25 km2
11.68 km2

= 46.72 cm2 : 11.68 km2


46.7 cm2
(1 d.p.)
Area on map which represents the airport = 46.7 cm2
3.

6 printed pages

Sum of exterior angles in n-sided polygon


26 + 31.5 + 43 + 55.5 + (n 4) 8.5
156 + 8.5n 34
8.5n
n

=
=
=
=
=

360
360
360
238
28

1/6

4.

(a) 20a3 4a2b 5a + b = 4a2(5a b) (5a b)


= (5a b)(4a2 b)
(b) 9x2 12x 32 = (3x 8)(3x + 4)

5.

(a)

=
=

Let x be the length of the rectangle.


x2 + 82 = 172
x 2 = 172 82
x = 17
8
= 15 cm
Area of rectangle = 15 8
= 120 cm2

8.

Volume of 7 spheres = 7

Using trial & error


factorisation

(2)3 = 8

Volume of pyramid = 791.681 cm3


Base area Height = 791.681

72 Height = 791.681
Height = 32.986
33.0 cm (3 sig. fig.)

=
(b)

9.

(a)

(x + y)
(x + y)2
x2 + 2xy + y2
x2 + y2
x2 + y2
x2 + y2
x2 + y2

(b)

(x y)2 =
=
=
=

=
=
6.

y =
Sub y = 2, x = 8,
2 =
16 = k
y =

r3

= 7 (3)3
= 791.681 cm3

=
=

7.

If y is inversely proportional
to x, then y = , where k is a
constant and k 0.

=
=
=
=
=
=
=

7
72
49
49 2xy
49 2(18)
49 + 36
85 (shown)

x2 2xy + y2
85 2(18)
85 + 36
121

2/6

10.

13. (a) Arc AB = 2

P = M+

= 18.3
18 cm (2 sig. fig.)

P = M 1
M =

(b) Areaof the major sector AOBC = 3.142 7.52


= 108.0062 cm2
108.006 cm2 (3 d.p.)
14. (a) 115 6900
1 60
75 60 75 = 4500
4500 Europeans visited the exhibition.

M =
M = P
= P
M =

(b) x = 360 115 75 125


= 45 (angles at a point)

Sub P = 4 and R = 2, M =
=2
11. (a)

7.5

PS2 + PR2 = RS2


PS2 = RS2 PR2
PS = 34
30
= 16

Using Pythagoras theorem

=
(ii) P(the visitor is European) =

(b) RP2 + PQ2 = QR2


QR2 = 302 + (16 + 24)2
QR = 30
16 24
= 50

12. (a) Mean score =

(c) (i) P(the visitor is Japanese) =

=
(iii) P(the visitor is Malaysian) =
=

=
= 11.1
(b) 9, 9, 9, 10, 10, 10, 10, 11, 11, 11, 11,12, 12, 12, 12, 12, 12, 13, 13, 13
Median score = mean of 10th and 11th score
=
= 11
(c)

Modal score = 12

3/6

SECTION B
15. (a)

(b)

17. (a) Least value of y x = 8 7


= 15

3(2x2 1) =
6x2 3 =
6x2 7x 3 =
(2x 3)(3x 1) =
or
x =

7x
7x
0
0
x =

=
18.

=
2
x + 8y
x
3(x 2) + (y 1)

=
=
=
=

2
10
10 8y
0

Sub (1) into (2),


3(10 8y 2) + (y 1) = 0
30 24y 6 + y

= 0

23 23 y = 0
23 y = 23
y = 1
Sub y = 1 into (1),

y = kx2
When x = 4, y = 16k
When x = 2, y = 4k

If y is directly proportional to x,
then y = kx, where k is a constant
and k 0.

Given that the difference is 36,


16k 4k = 36
12k = 36
k = 3
y = 3x2

16.

(b) Greatest value of

Multiply 4 on both sides


(1)
(2)

When x = 1,
y = 3(1)2
= 3
19. (a) Given the mode is 2, x > 12.
Smallest possible value of x = 13
(b) Mean =

=
= 1.294
1.29 (3 sig. fig.)
Median = mean of 17th and 18th values
=
=1

x = 10 8
= 2

4/6

23. (a) p = 1.5, q = 1

20. (a) Modal travel time = 35


(b) Median travel time = mean of 14th and 15th values
=
= 36

x
y

2 1
5 1.55

0
1

1
2.5

2
3

3
2.5

4
1

5
6
1.5 5

(b)

(c) Mean travel time taken by students from home to school





=

=
= 36.4
21. (a)

p
q
r
s

=
=
=
=

1111155556
111111555556
31
37

(b) Sum of digits for 3333333342 = 7 + 8(6)


= 55
(c) Sum of digits in the Q column in the nth row = 6n + 1
22. (a) A = {3, 5, 7, 11, 13, 17, 19}
(b) B = {3, 5, 7, 9, 11, 13, 15, 17, 19}
(c) A B = {3, 5, 7, 11, 13, 17, 19}

AB
The intersection of sets A and B
is the sets of elements which are
common to both A and B

(d) A is a subset of B.
(e) A B = {3, 5, 7, 9, 11, 13, 15, 17, 19}
AB
The union of sets A and B is the
set of elements which are in A, or
in B or in both A and B.

Each member of A is also


member of B, A is a subset of
B
(c) (i) y = 1.85
(ii) 1.45, 5.45
(d) x = 0.4, y = 1.70 or x = 4.9, y = 1.30

5/6

24. (a) Slant height of cone = 5


12.3
= 13.27 cm

Using Pythagoras
theorem

Total surface area of the cone = rl + r2


= 5 (13.37) + (5)2
= 287.1138
= 287 cm2 (3 sig. fig)
(b) Volume of cone = r2h
= 3.142 52 12.3
= 322.055 cm3
Volume of water = 3.142 102 12.3 322.055
= 3542.6 cm3
3540 cm3 (3 sig. fig.)

(c) Final depth of the water =


=

= 11.275
11.3 cm (3 sig. fig.)

6/6

You might also like